User:Nil Einne/Sandbox/RDS/Recent

From Wikipedia, the free encyclopedia
Welcome to the recent Wikipedia Science Reference Desk archives
The page you are currently viewing contains recently archived questions. While you can leave answers for any questions shown below, please ask new questions on the current Science Reference Desk page.


October 27[edit]

QED[edit]

Surely quantum electrodynamics was so named as a play on Q.E.D.. Is there a nice succinct quote about this from the 1920s or so when it was being developed? Comet Tuttle (talk) 00:35, 27 October 2009 (UTC)

Mm... was it? I have never seen that claimed, even in books on the subject. --Mr.98 (talk) 00:59, 27 October 2009 (UTC)
Ask Feynman the next time you see him. DRosenbach (Talk | Contribs) 02:04, 27 October 2009 (UTC)
I think that's just a coincidence. Dauto (talk) 03:39, 27 October 2009 (UTC)
I would not put it outside of the realm of possibility; after all these sorts of people are the same that gave us "strange" and "charm" for names of quarks; heck the name "quark" itself was intentionally chosen because it resembled a duck's quack and a seagull's call. They also gave us the name "gluon" because they "stick things together" (i.e. "glue"-ons). Physicists have their own brand of wit, so the connection between "quantum electrodynamics" and "quod erat demonstrandum" may have been more than a happy coincidence. I have no citations to say one way or other; I just would not be surprised if it were so... --Jayron32 03:39, 27 October 2009 (UTC)
Feynman definitely intended the parallel in his book. --antilivedT | C | G 04:19, 27 October 2009 (UTC)
I thought quarks were so named for "Three quarks for muster Mark" - a phrase from Finnegans Wake by James Joyce? In other words, as there were three flavours of them, something that came in threes were quarks... A quote to this effect from Murray Gell-Mann,the discoverer of quarks is given under Quark#Etymology .--TammyMoet (talk) 10:36, 27 October 2009 (UTC)
Joyce was using the "three quarks for Muster mark" in reference to three honks of a seagull; while Murray Gell-Mann used the term "kwork" in reference to a duck's quack. So he specifically picked the spelling from Finnegan's Wake since he liked the spelling better but ALSO had roughly the same meaning as his first word. --Jayron32 13:26, 27 October 2009 (UTC)
I don't put it outside the realm of possibility... I just haven't heard anyone claim it. They were using "Quantenelektrodynamik" as a name for it when Feynman was still in diapers, he doesn't have anything to do with the name... similarly, "quark" and its names are from a much later generation (the 1960s). Were Dirac and Pauli as funny with their physics? Not generally... --Mr.98 (talk) 14:40, 27 October 2009 (UTC)

Special Relitivity[edit]

If the speed of light through space is constant, regardless of the movement of an observer...wouldn't an individual be able to measure his movement through space (the combined velocities of the earth rotation, earths orbit, sun's movement through the galaxy, galaxy though universe, etc.) by the difference in the apparent speed of a light emitted by the user in various directions? That is, assuming that all of the above movements were lined up in the same direction at the time of measurement, wouldn't light appear to move faster from the observer when a beam is aimed to (for example) the West then when aimed to the East? I would think that the combined velocities would give a vector and velocity for the observer, and that it would be detectable by measuring the apparent speed of light (produced locally) from the perspective of the moving observer. I am referring to a local observation. I understand this has been seen as a red shift when observing incoming galactic light from a galaxy that is moving away from the observer. A Glass Bubble (talk) 00:52, 27 October 2009 (UTC)

No, the speed of light is, as you say, constant regardless of the movement of an observer, so it is exactly the same in all directions. --Tango (talk) 00:56, 27 October 2009 (UTC)
What Tango said. For more information, you might get some insight from Velocity-addition formula. You can also read about some of the bizarre consequences of this in Relativity of simultaneity. Someguy1221 (talk) 01:00, 27 October 2009 (UTC)
Got it, and I will go read the links (Thanks)...and not to be thick headed...but if it is the same speed in all directions through space, and I as an observer am moving (for the above reasons). Shouldn't I be able to detect that a beam shot from me towards my direction of movement is moving away from me (key from me) slower than if I turned around and shot a beam away from my direction of movement? I understand that an observer at close to the speed of light will start seeing a shift in the appearance of light directly approaching him, and warping of the light to his sides. I thought that this might be a theoretically detectable phenomenon at the velocities that you and I are traveling at.A Glass Bubble (talk) 01:04, 27 October 2009 (UTC)
I think you're still misunderstanding. It is simply that no matter how, when or where you measure the speed of light (in vacuum) - you always get the exact same answer. So, for example, if you measure the speed of the light coming from a far distant star in summer when (say) the earth is moving away from it in our orbit - and then again in winter when the earth is moving towards that same star - the speed of the light you measure coming from the star is exactly the same. Even though the relative motion of the earth was 100,000 km/hour away from the star in the first measurement - and 100,000 km/hour towards the star in the second measurement - so you might expect your instrument to read 200,000 km/hour different in the two cases - it won't - you'll get the exact same answer. Similarly - if we measure the speed of light from something like a comet when it's rushing towards us - it's exactly the same as when it's moving away. No matter what you do - any measurement of the speed of light in vacuum will produce the exact same numerical result. Light it quite different from everything else in that regard. If you measure the speed of sound waves or water waves or the speed of an oncoming car - it'll depend on how fast you are moving and perhaps on how fast the source was moving - but light is completely weird in that regard. SteveBaker (talk) 14:06, 27 October 2009 (UTC)
Strictly speaking, there's no real way to test the constancy of the speed of light with respect to special relativity. That is to say, special relativity doesn't hypothesize that the speed of light is the same to all observers, but rather it assumes as much. The real test then, is in all the other predictions made by the theory, as they all rest on this assumption. And indeed, special relativity has proven remarkably accurate in experiment after experiment, except in those that require the use of general relativity. But there were attempts to see if the speed of light varied with the reference frame, and you can read about those at Michelson–Morley experiment and Fizeau experiment (well, that second experiment is not quite about the reference frame, but still related). Someguy1221 (talk) 01:12, 27 October 2009 (UTC)
These questions get at the heart of some of the major problems with pre-relativity physics, and were the motivations for the development of special relativity. I suggest looking for a good introduction to topic to get an understanding of how exactly relativity deals with these problems. You could try Introduction to special relativity, but I'm not really sure how clear that article is. Googling "Introduction to special relativity" comes up with a bunch of stuff. Rckrone (talk) 20:16, 27 October 2009 (UTC)
It involves the warping of space and time. To put it simply, if you had two different observers moving at two different velocities looking at a snapshot of photon that was moving towards them, they'd both agree on how fast it was going, but not on how far away it was or even when the picture was taken. — DanielLC 05:18, 28 October 2009 (UTC)

Underdamped Harmonic Oscillator[edit]

I have trouble understanding how damping affects the period of an underdamped harmonic oscillator. I know that damping affects the amplitude of the oscillator, however, how would damping change the period (or frequency)?

I feel like the period should be unaffected because the angular frequency, omega, equals sqrt(omegazero^2-beta^2), which seems to me is a constant.

Note: omegazero equals the natural (undamped) angular frequency, and beta equals the damping coefficient.

Thanks in advance for the help!

Steevven1 (Talk) (Contribs) (Gallery) 01:21, 27 October 2009 (UTC)

As you said, the angular frequency is sqrt(ω022), which is less than ω0, so the oscillation with damping is slower than without damping. The angular frequency is constant in time, which means that the period doesn't change while the oscillator is running, even as the oscillations become smaller. But it's not constant in β, which means changing the amount of damping will affect the period (more damping means longer period). Rckrone (talk) 02:44, 27 October 2009 (UTC)
Thanks for the response, but can you explain how beta is not constant? Thanks again. Steevven1 (Talk) (Contribs) (Gallery) 06:01, 27 October 2009 (UTC)
β is constant for a particular oscillator. Your question was, if β is changed, will ω change? and the answer is yes. For example, suppose I have an undamped oscillator (β = 0) with an angular frequency of 1.0 rad/sec. Now I add a damping force with coefficient β = 0.6 rad/sec. The angular frequency now is sqrt((1.0 rad/sec)2 - (0.6 rad/sec)2) = 0.8 rad/sec. That's slower than it was without damping. Rckrone (talk) 19:12, 27 October 2009 (UTC)

The article Harmonic oscillator provides the equations for the damped case. Not using the beta symbol the underdamped frequency is where ζ is a constant called the damping ratio.Cuddlyable3 (talk) 12:36, 27 October 2009 (UTC)

GMO hemp[edit]

With all the other GMO corn, rice, peanuts, etc. why not GMO hemp to get rid of the THC so hemp can be grown once again (in the US legally) and provide us with abundant fiber, food and fuel? 71.100.9.185 (talk) 04:52, 27 October 2009 (UTC)

Because there are other, less expensive, and better sources of fiber, food, and fuel than hemp. The whole "legalize hemp because it can be used for really useful things" is usually a cover for the fact that the most useful thing it can do is get you high. While I will not deny that hemp has been quite useful for many applications, it is not more useful than other substances for the same applications, which makes genetically engineering a THC-free hemp kinda pointless. For more reasons than one. --Jayron32 05:18, 27 October 2009 (UTC)
And what might those other reasons be? ...hemp grows virtually anywhere, can be grown and used to make fiber food and fuel easily by the grower - cutting out the middle and the end man, would end American dictatorship and restore the democratic right of Americans? ...and probably a lot more. 71.100.9.185 (talk) 06:31, 27 October 2009 (UTC)
You're really on a roll with your conspiracy theories recently. Here's a suggestion for you - find someone who's big into marijuana sorry, hemp legalization and argues the "we can make stuff with it!" point, then invite them to join you with a placard declaring your support for THC-less hemp bioengineering. I strongly suspect all of your vocal proponents will suddenly be nowhere to be found... because what they want is legal pot to smoke, not an alternative to cotton. 218.25.32.210 (talk) 07:37, 27 October 2009 (UTC)
Yes, still a conspiracy but this time on the part of potheads ...er people who like to wear hemp ...er because it does not have the same depressing connotation as cotton has with slavery. Get it? COTTON = SLAVERY, HEMP = FREEDOM. Freedom, man freedom. That's what its all about. 71.100.9.185 (talk) 11:05, 27 October 2009 (UTC)
Paper made from hemp being considerably more durable and longer lasting than wood pulp doesn't match Jayron32's description of the product being less useful.163.1.147.64 (talk) 12:25, 1 November 2009 (UTC)

Interestingly enough, hemp might be a good target for pharming use anyway, since there shouldn't be any native crops which could be cross-contaminated. The cultivation techniques are well known, and the plant's toxicity is, as far as I know, limited to the THC, which is a well-characterized and, in the grand scheme of things, well tolerated chemical. SDY (talk) 08:00, 27 October 2009 (UTC)

"why not GMO hemp to get rid of the THC?" => Most other sources of fiber rot (e.g. flax). Thanks to the THC hemp does not. DVdm (talk) 11:34, 27 October 2009 (UTC)

Interesting read on this topic : Is hemp (nonpharmacological marijuana) the answer to our environmental problems? APL (talk) 13:40, 27 October 2009 (UTC)
Ah, the complication for law enforcement. Who hasn't had someone who smoked pot do a 180 on you and if the are the skitziod type reveal their Jekyll personality? I say keep marijuana illegal except for use by qualified psychiatrists to treat psycho patients in the psyco ward or in their office. —Preceding unsigned comment added by Biggerbannana (talkcontribs) 16:04, 27 October 2009 (UTC)

As mentioned in Hemp, it is already legally grown in a number countries throughout the world. I believe that list is incomplete anyway it appears there are a number more countries from (old and not exactly a neutral source but no evidence it's inaccurate) although it's possible some of them it's more of a technically legal issue like the US rather then actually done in practice kind of thing. And as also mentioned there and elsewhere there are already varieties of hemp with low THC, according to this below 0.05% [1]. The EU evidentally allows anything below 0.3% while NSW allows below 1% [2]. Also from what I can tell from the article and [3] [4][5] [6] the situation in the US is unclear. It is legal in some states. Federal law technically allows it with special requirements including a Drug Enforcement Administration permit/license, but they've refused to issue them. It appears from my earlier ref this is being tested in court so the legal situation may be cleared up in the future. (Or maybe Obama will change things or stuff like this [7] will compel changes or Congress wil change htings [8].) In any case, I'm doubtful that the minimal amount of THC in hemp is the reason why some governments (including the US) are reluctant to allow hemp to be grown. Much more likely is because of the difficulty actually differentating varieties with a low THC and a high THC. As police in the Netherlands found out [9] when they destroyed a universities' crop. Clearly allowing anyone and everyone to grow hemp wherever they want is likely to be problematic if you want to prevent marijuana/cannabis varieties with high THC being grown. That's likely why in many countries were it is allowed it's still fairly regulated. But these countries trust their ability to ensure only low THC varieties are grown, the US Federal government or specifically the DEA apparently does not. Completely eliminating THC from hemp is not likely to help this. There's no real solution other then improving testing methods and convincing the government it is a practical to regulate it. Sure you could do something like make GFP hemp but illegal growers could likely replicate that presuming they can't just breed it in to their existing cultivars so it's not likely to be a useful strategy. Since the most interest in hemp appears to come from people like you who don't like GM plants, I also question the merits of even going down the GM route beyond perhaps proof of concept. BTW, who said that hemp isn't associated with slavery? [10] [11] (yes I'm aware it says slavery wasn't essential). Edit: Actuallying considering [12] [13] List of Schedule I drugs (US) Controlled Substances Act and some of the earlier refs, I may be mistaken about the THC thing. I don't really know much about US law and I often find some of the stuff they do strange particularly when it comes to the interaction of state and federal law. It's possible that the reason why the DEA limit (or claim they can limit) the growth of hemp is because it contains THC which is a controlled substance therefore it you completely eliminated THC the DEA could not claim such a right. However given some of the refs particularly the food stuffs issue, I'm not convinced the DEA's claim is legit and it seems possible if there really is desire to control hemp the absence of THC is ultimately not going to stop them since the law could potentially be amended to control hemp even if it doesn't contain THC. Nil Einne (talk) 21:59, 27 October 2009 (UTC)

Maybe this example of hypocrisy being practiced by the US government would be the last straw for the American people... Biggerbannana (talk) 05:40, 28 October 2009 (UTC)
I'm surprised no one has mentioned the conspiracy theory-like but not totally crazy notion that DuPont worked to make hemp illegal, or nearly so, in order to make their nylon product more dominant. There's a brief mention of it at DuPont#Hemp. Pfly (talk) 09:14, 29 October 2009 (UTC)

Tsar Bomba[edit]

According to our article on the tsar bomba, it states, "In terms of physical destructiveness, much of its high yield was inefficiently radiated upwards into space". Is this true? In addition, I did some research and there are concepts and designs for nuclear shaped charges that can direct the energy released by a nuclear explosion into a cone as small as 22.5 degrees. Would a shaped charge create additional ground damage if used in a similar high yield bomb? If so, how large should the cone be and at what altitude should it be detonated at to maximize ground damage if we assume the bomb is 50 megatons in yield. ScienceApe (talk) 06:21, 27 October 2009 (UTC)

The problem here may be in ft/sec propagation of the blast wave front. The main advantage of a shaped charge is to make the copper cone lining into a plasma and while so doing propel it more or less as a small diameter cylinder. A very high ft/sec blast wave might not allow for the cylinder to be formed properly or propelled in a linear or even cone shaped fashion. 71.100.9.185 (talk) 06:38, 27 October 2009 (UTC)
The nuclear shaped charge is described here, http://www.projectrho.com/rocket/rocket3x.html#shapedcharge and here as well http://arxiv.org/PS_cache/physics/pdf/0510/0510071v5.pdf ScienceApe (talk) 07:44, 27 October 2009 (UTC)
Again the problem with this is the power and speed of the blast. This thing would have to be as big as a stadium to achieve anything besides everything turning to plasma closer to the center of blast. 71.100.9.185 (talk) 11:36, 27 October 2009 (UTC)
According to those designs, the materials are supposed to turn into plasma. ScienceApe (talk) 15:53, 27 October 2009 (UTC)

I'm pretty sure that the article isn't referring to shaped charges. My guess is that what it's talking about is the high detonation altitude (4 km), since the Soviets really didn't want to blow away a lot of ground and create a lot of fallout. The fireball from the explosion was ~4.6 km, so they basically dropped it at the ground and missed. SDY (talk) 08:05, 27 October 2009 (UTC)

That really wasn't what I was asking. I know it wasn't shaped. ScienceApe (talk) 15:53, 27 October 2009 (UTC)
You can see from the Tsar Bomba videos that it was not shaped in any way—it was just a perfect sphere. The shaped charges used for Project Orion are miniscule in comparison to the Tsar Bomba. You could probably shape an arbitrary-sized explosion but the size of the casing and liner needed to shape it gets prohibitive—you aren't going to fit the resultant bomb onto an airplane. Anyway, what's the point of such a thing? Even the 50MT was an active of political demonstration, not a feasible weapon. There are basically no targets where a 50MT is better than a few 5MT, which are much more manageable. As you probably know, increasing yield of individual weapons generally just increases the wasted energy. --Mr.98 (talk) 14:17, 27 October 2009 (UTC)
Yes, I know the Tsar Bomba wasn't shaped. I know what the Tsar Bomba's intention was, and its impracticality, and I know 5MT bombs are more manageable. However, I simply want to know if a shaped charge would in fact increase the ground damage, and if so, what would the cone have to be and at what altitude it should be dropped at. Your last comment is what I'm after. If shaping a charge will reduce the energy being wasted in a large explosion. ScienceApe (talk) 15:53, 27 October 2009 (UTC)
I don't see why shaping it would not reduce the energy being wasted, since most of it would no longer be directed away from your target (as with the sphere, the top half basically just goes up, where there isn't anything to hit... maybe some of that reflects off of the atmosphere and comes back down again, but it's not very much). The problem, again, is that shaping 50MT is probably an extremely non-trivial task. Anyway, there's a question to me of whether "wasting" is really the right terminology here -- the goal is not really total efficiency, but a question of how much of a yield you want in one place at one time. Anyway, I understand that this is peripheral to your question. --Mr.98 (talk) 20:43, 27 October 2009 (UTC)
strange diversion on the topic of the morality of the free exchange of ideas. (I think.)

Freedom of useful information like the above is a wonderful thing. What is an efficient way of raping a person? Cuddlyable3 (talk) 12:23, 27 October 2009 (UTC)

Except that the barrier between someone raping someone else and someone building a hydrogen bomb is totally different. (Maybe seven nations have developed hydrogen bombs, ever. Compare that to how many rapists there are.) Only advanced nuclear states have the materials to build a hydrogen bomb (which requires already having a fission bomb, and sufficient understanding to use it in the most optimal way), and they aren't going to spend their billions on something they found on the internet. Even then, there is little evidence that suppressing information actually suppresses unwanted illegal activity—rather, many have argued that if more information is out there, more people know what sorts of unquestionable behavior to look for. --Mr.98 (talk) 14:10, 27 October 2009 (UTC)
I agree that the scale of the assaults is different when seen by the perpetrator, but whether they are so different when suffered by the victim depends on the victim's distance from ground zero. I suggest that states or organisations who are interested in leap-frogging nuclear weapon capability in fact do check Wikipedia as one of their sources of information. Responders knowing that can answer appropriately to the OP's request for bomb-making information. Cuddlyable3 (talk) 19:46, 27 October 2009 (UTC)
Er, no, it is not about the scale of the assaults, it is about the likelihood of them happening in the first place. Talking about h-bombs in a speculative way isn't going to help or hurt anybody, presuming that nobody on here actually works on them for a living. (And even then, I think one would find that what counted as "useful" information is far more technical than you and I would recognize.) --Mr.98 (talk) 20:43, 27 October 2009 (UTC)
I'll get to work on adding high-energy physics to the list of things we can't answer questions about, but for the moment you might enjoy the US Government's 1965 publication The Los Alamos primer: the first lectures on how to build an atomic bomb or if you're really dedicated to the Super, check out The H-Bomb Secret, which the US was rather less happy to have published in 1979. --Sean 21:27, 27 October 2009 (UTC)
Both nuclear bombing and rapes have happened. One cannot presume that either will not happen again. The OP has researched how to carry out the former and seeks numerical information to maximise the ground damage which is where people will be hurt. Judging by the links cited about shaped charge the OP's research has found the most detailed information that is publically accessible. There are certainly responders here who are qualified to add to that information. The free-access principle of Wikipedia means they cannot know who may use its public information. We live in a time of Nuclear proliferation where quantitative engineering details for making a nuclear WMD have to be limited by Need to know not "cool to know". Sean thank you for fascinating references. The Los Alamos Primer is part of mostly classified material whose historical significance is undeniable. High-energy physics encompasses much more of importance than how to destroy people so I don't see a good reason to ban that subject. Cuddlyable3 (talk) 22:06, 27 October 2009 (UTC)
First of all, as others mentioned above, a 50-megaton bomb has no uses outside of political saber-rattling, and shaped charges are not helpful for that. Second, shaped charges already exist. We're discussing their capabilities, not offering improvements to existing designs. Third, if you seriously think that any terrorist who can make a nuke doesn't already know EVERYTHING that we can possibly discuss here like the back of their hand, you're severely underestimating the difficulty of making a bomb. --99.237.234.104 (talk) 01:25, 28 October 2009 (UTC)
The whole objection is an absurdity resting on the premise that the basic idea of a piece of repurposed artillery shooting one subcritical mass at another is so insufficient a weapon for terrorists and rapists that they must resort to the Wikipedia chat room to get ideas for maximizing their body counts. --Sean 01:52, 28 October 2009 (UTC)
The Ref. Desk is not a chat room. Shaped charges are uninteresting to rapists. Limited access to fissionable material is one of the few difficulties left for rogue states and terrorists seeking to make a nuke. Technology such as shaped charges that are key to detonating a subcritical mass must be highly attractive to them. Look here. Every bad guy has an Internet connection. It would be naive to suppose nobody in emerging and wannabe nuclear-capable groupings Iran, DPRK, Al-Qaeda or any of these learns nothing from Wikipedia or the links so freely given here. That means us. Shaped charges certainly exist. Their quantitative design data for WMD use is neither in Wikipedia mainspace, Popular Mechanics or otherwise in public domain because this is on-going research data from Nuclear weapons testing. Sean you provided aerial views of Nagasaki that don't show its inhabitants. Here is one of the lucky ones. Cuddlyable3 (talk) 08:47, 28 October 2009 (UTC)
Not a chat room? You started it! In fact you , frequently start it. You're always first to jump in with a joke, even when the question isn't answered. Collapsing this off topic stuff.APL (talk) 12:51, 28 October 2009 (UTC)
Please ask on Talk page if you remain uncertain whether the Ref. Desk is a chat room. No joke was added to this thread so your reference to jokes is a comment on a contributor not a subject. I have made visible your "hidden" questions about rape since that allows them to be treated properly and hope you don't mind. I agree that there are several parameters for quantifying rape efficiency depending on the rapist's motivation. In analogy to the tsar bomba one's goal could be to cause maximum human devastation and noise for the purpose of intimidating others, inflicting disease(s) would be an acceptable secondary achievement. You should not ask a contributor's gender. Cuddlyable3 (talk) 11:14, 29 October 2009 (UTC)

spin forbidden versus classically forbidden[edit]

So I did some weird light reaction with benzophenone. So in exciting from pi to pi-star, evidently fluorescence is kind of slow because the pi star electron, if we were to follow Schrodinger's equation classically, would have to go through a point of zero probability (the nucleus) in order to make the transition, right? So some quantum tunneling must occur.

However, in lab we also have "spin forbidden" reactions -- I assume this is a different kind of forbiddance? What is the difference between spin forbidden and classically forbidden? What is spin forbidden anyway, in layman's terms? (Or terms for a second year undergraduate.) Thanks! John Riemann Soong (talk) 08:58, 27 October 2009 (UTC)

SPin forbidden reactions would be ones that caused two electrons of the same spin to occupy the same orbital. I can't think of any off the top of my head (its been too long for me since I did this stuff), but there may be some transition states that cause the creation of such orbitals. --Jayron32 13:10, 27 October 2009 (UTC)
It's a bit more complicated than that. For a simple (that is, non-relativistic) case, which means for electrons bound to light atoms like C and H, the wavefunctions are products of spatial part and spin part. The electric dipole operator only acts on the spatial part. Thus, when spin part of the wavefunction of the electron that makes the transition is different in the initial and in the final state, the electric dipole operator matrix element between the initial and the final state is zero, and the transition is spin-forbidden. Now, I am deliberately oversimplifying it. For a really thorough explanation of the Selection rules for electron transitions in atoms and ions, see Theory of Atomic Structure and Spectra by Cowan. There must be a similarly voluminous textbook for transitions in molecules, too, but I am less familiar with the molecular transitions. The basis of the selection rules is the same in atoms and in molecules, of course. --Dr Dima (talk) 18:34, 27 October 2009 (UTC)

Black worms with antenna???[edit]

Hello, I have had, just in the past month or so, small black worms (with antenna), crawling on my garage floor. They seem to just crawl in and die on the floor ( although one was crawling on the wall). I have just been vacuuming them up with my shop vac, but in a day or two, I have to do it again. I live in West Michigan. I am wondering (1) what are they and, (2) would they get into anything like the cars that are parked there?

I have been trying to find out what they are and, to this point, have had no luck!

Thank you - Dan —Preceding unsigned comment added by Danc8770 (talkcontribs) 13:01, 27 October 2009 (UTC)

Do they have any other coloration (e.g. red spots)? Do they have legs? (How many?) They sound like larvae of some sort, not worms, per se, but without a picture it's hard to make heads or tails of it. --Mr.98 (talk) 14:07, 27 October 2009 (UTC)
Maybe woodlice? Icek (talk) 19:33, 30 October 2009 (UTC)
As Mr.98 says, much more details, or a picture, would be necessary for a meaningful answer. My own best guess would be millipedes or centipedes: there are around 10,000 different species of the former and 8,000 of the latter. They might also be the caterpillars of some local butterfly or moth; these do not have antennae (unlike the adult insects), but some have spines, horns or tufts of hairs that may be mistaken for antennae. To answer question (2) - if they can climb a wall (as all of the suggestions so far can) then they might be able to get inside a parked car (by climbing the tyres or dropping from above), but would probably have little motivation to do so. 87.81.230.195 (talk) 06:46, 1 November 2009 (UTC)

disaster preparesdness[edit]

1.please tell me something about disaster preparedness fr eg: first-aid kit what to keep in it etc please ans soon thank you —Preceding unsigned comment added by Advaitadd (talkcontribs) 13:51, 27 October 2009 (UTC)

You might check out our article on bug-out bag for one potential list. The answer depends on what kind of disaster you are preparing for—there's a sweet spot between "electricity goes out for a day" and "end of civilization" that most people are aiming for, given that the former doesn't really require more than a flashlight and some batteries, and the latter is not something you have much of a chance of surviving anyway. What you probably want, ideally, is "keep me afloat for a few days so that I can get the heck out of here if things are really going sour," with modifications depending on your particular local risks (e.g. whether you are on a flood plain vs. in an earthquake zone vs. wildfires, etc.). We also have an article on survival kits more generally—but these, IMO, are assume you are really going to survive long-term in such a situation, which I personally find unlikely. (Flee to higher ground, find help! That's about all I think one should be trying to do, not eke out some sort of The Road lifestyle.) --Mr.98 (talk) 14:59, 27 October 2009 (UTC)
I think you need to consider the kinds of disaster you are preparing for. If (for example) you live in a small town in tornado alley and you want to prepare for tornado season - then you need a good first aid kit - enough so you can help people out in the hour it might take for help to arrive. But if you were involved in the aftermath of hurricane Katrina - with the possibility of it taking an entire week for help to arrive in enough quantity - then food, bottled water - a way to hack your way out of your attic if you're trapped there - maybe a small inflatable boat. It really does depend on what kind of thing you think might happen. Consider also where you're going to be. You need loads of bottled water for surviving here in Texas - but perhaps that's not a consideration in the wilds of Alaska. Of course if you're planning some kind of extreme travel (trekking across Death Valley, skiing in the Antarctic, taking a mini-sub into the Marianas Trench) - then your plans for disaster will be MUCH more specific!
I would argue that from a day-to-day perspective, the Katrina type of problem is at the more extreme end of the scale. It's very rare for help not to arrive within hours to days in most places in the civilised world. In which case, perhaps you should consider things like a safe, waterproof place to keep important documents (copies of insurance policies, deeds for property, bank account details, etc) - pre-paid credit cards - some cash. Things that will help you when you're kept out of your home (or what remains of it) for weeks or more.
Survival experts often quote the following rules of thumb:
  1. Humans cannot survive more than three hours exposed to extremely low temperatures.
  2. Humans cannot survive more than three days without water.
  3. Humans cannot survive more than three weeks without food.
So should you keep food in your emergency stores? Perhaps not. If you're not going to be rescued within three weeks - then you're in a lot of trouble! If we have to survive cold or wet weather then warm clothing might be more important. Will you need to make a shelter? A decent multi-tool (a Swiss Army knife or a Leatherman multi-tool) will give you huge advantages for a tiny amount of space/weight in your kit. A way to make fire (waterproof matches). Maybe a waterproof sheet and some rope to make an improvised shelter - a mylar foil blanket to keep yourself warm & dry. A first aid kit certainly wouldn't hurt - you should have one of those in your home and car anyway. When you have all of those immediate needs taken care of - then bottled water is your next priority - and only then consider some long-life food. Trail mix, jerky, energy bars.
SteveBaker (talk) 02:44, 28 October 2009 (UTC)

Make up of the Brain[edit]

What percentage of the brain is water, fat and protein? Generally people and sites quote: 70 or 76% is water, 60% is fat and 40% is protein. But, that adds up to over 100%, so what am I missing?Clb14a (talk) 15:25, 27 October 2009 (UTC)

Without knowing the answer, those numbers suggest that the 60 and 40 percent numbers exclude water. --Sean 19:34, 27 October 2009 (UTC)
A lot of weights in biology are measured dry - the water is driven off before measurement. Vimescarrot (talk) 20:58, 27 October 2009 (UTC)
This says that the makeup is 77 to 78% water, 10 to 12% lipids (fat), 8% protein, 1% carbohydrate, 2% soluable organic substances and 1% inorganic salts. That article is well referenced (those particular numbers come from: McIlwain, H. and Bachelard, H.S., Biochemistry and the Central Nervous System, Edinburgh: Churchill Livingstone, 1985) - so I think we can take these numbers are true. So if you remove the water, then the remainder is reasonably close to 60% fat, 40% protein - so I think TotoBaggins and Vimescarrot have nailed the reason for the confusion in what you read. SteveBaker (talk) 02:22, 28 October 2009 (UTC)

destuctive interference[edit]

In water, suppose a crest and trough of equal magnitude travel towards each other and causes destructive interference, how is that, a short while later, that the crest and trough each *pop* out again and continue their own merry way as it will be if they had not met in the first place? Where did the *information* go during the destructive interference? another dimension?

does the same thing happen with matter and antimatter too? —Preceding unsigned comment added by 118.100.10.187 (talk) 15:34, 27 October 2009 (UTC)

With simple waves neither a crest nor trough travel unaccompanied. If two waves did perfectly interfere then they wouldn't carry on their merry way, they cancel out. --Digrpat (talk) 18:15, 27 October 2009 (UTC)
That's not right. Destructive interference is simply the superposition of the two waves. So wave packets traveling toward each other will effectively "pass through" each other, like the OP says. Red Act (talk) 19:06, 27 October 2009 (UTC)
No additional dimensions are involved. The "information" at the instant that the water is flat is in the kinetic energy, or equivalently the momentum or the velocity of the water, depending on how you want to look at it. The potential energy of the gravity wave, which is determined by the position of all of the water molecules, may for an instant be as if the water was static. But in order to completely describe the water at that instant, you also have to describe the velocity of each of the water molecules. (There are also additional complexities involved due to water not consisting of point particles, but I don't think it's important to bring that up here.)
The annihilation of matter and antimatter is different, in that the annihilation process produces new, different particles. And there isn't an instant of time at which the old particles don't exist anymore, and the new particles don't exist yet. Once you get down to the level of quantum mechanics, you can't say exactly what's happening at a precise instant like that, because the Heisenberg uncertainty principle requires that an infinitely large amount of energy would need to be involved, in order to know exactly when something happened. Red Act (talk) 19:06, 27 October 2009 (UTC)

Why do most humans grimace when they're straining themselves?[edit]

Back off!

Such as when they're trying to lift something heavy or during a very large bowel movement? 20.137.18.50 (talk) 15:36, 27 October 2009 (UTC)

Well it could be evolutionarily beneficial because those heavy things may have been a predator or other human, and animals tend to show their teeth to warn attackers that they can bite. --Mark PEA (talk) 18:29, 27 October 2009 (UTC)
It's simply a result of tensing all the muscles of the face and jaw -- when you strain, pretty much every muscle in the body contracts to some degree. Looie496 (talk) 21:27, 27 October 2009 (UTC)
It seems rather unlikely that something like tensing your face while lifting something would exert any reproductive pressure on a population. "Ah, well, if Bob had only tensed his face while lifting that rock, he'd probably be alive by now..." We're getting into Just-so story territory here... --Mr.98 (talk) 01:42, 28 October 2009 (UTC)
It was just a speculative hypothesis, like much of evolutionary psychology. The fact is, a gene which makes one bare one's teeth when straining is more beneficial than one that doesn't, in the scenarios where the strain is caused by a predator/enemy on top of oneself. What isn't a fact is that natural selection favoured this, and Looie's answer is the much more likely explanation.
Prey... Has teeth-bare gene Has no teeth-bare gene
Is lifting heavy object -0.1
(Miniscule ATP loss due to grimacing)
+0.0
(No facial muscle-related energy loss)
Is lifting heavy predator +?
(Predator fears being bitten, runs away)
-∞
(Predator doesn't fear being bitten, kills prey)
's net utility: > -0.1 -∞
The net utility obviously assumes that both conditions occur, which isn't an unreasonable assumption for >10,000 years ago seeing as animal-human and human-human attacks still occur to this day. I know it also seems to assume that grimacing will cause the predator to run away 100% of the time, which is clearly not the case. I still think it is worth it even if a fraction of predators run away.
Feel free to pick away at any flaws, and remember that I'm being hypothetical. --Mark PEA (talk) 11:43, 28 October 2009 (UTC)
It's really tough to know - but I guess another possibility is that this grimacing is proto-language - like smiling, crying, frowning and laughing. Before the ancestors of humans learned to speak - they would have used facial expressions just like dogs wag their tails in different ways to signify welcome (circular wag) or happiness (side-to-side wag). Perhaps the grimace in the context of someone lifting something means "This is really heavy - could you please come over and help me?". SteveBaker (talk) 12:23, 28 October 2009 (UTC)
Or maybe it's because the muscles in your neck actually contribute to the lifting of heavy weights. Try lifting something heavy without grimacing and you'll see what I mean. Conversely, try exerting a large force with your legs, with your back braced against a wall, and you won't feel such a strong need to grimace - although the tension in your back will affect your face to a lesser degree. --Heron (talk) 18:00, 28 October 2009 (UTC)

Atherothrombotic Cerebral Infarction[edit]

What is this condition? What are the symptoms and treatments for this? --Reticuli88 (talk) 15:54, 27 October 2009 (UTC)

You should check out our Cerebral infarction article. You'll have to see Wiktionary for atherothrombotic. —Akrabbimtalk 16:13, 27 October 2009 (UTC)
The stroke article is fairly comprehensive and has detailed sections on symptoms and possible treatments. --- Medical geneticist (talk) 16:18, 27 October 2009 (UTC)

Oh! ok, I didn't know it was a type of stroke.--Reticuli88 (talk) 17:14, 27 October 2009 (UTC)

Athero would refer to a "blood vessel," thrombotic would refer to a "clod of platelets," cerebral would refer to the upper most, paired lobes of the brain known as the "cerebrum" and infarction refers to a "blockage of a blood vessel." DRosenbach (Talk | Contribs) 23:22, 27 October 2009 (UTC)

Triceratops And Their Frills (And Also Stegosaurus)[edit]

If a triceratops were to somehow fall onto its back, would it have a problem getting back upright due to its frill? How about a stegosaurus and its plates, would it also have a problem? -WarthogDemon 16:00, 27 October 2009 (UTC)

Well, barring the plates getting stuck in the soil from falling off a ledge, wouldn't they help it right itself, as the frill and plates would prevent a creature from staying on its back and would instead roll it to its side? ~ Amory (utc) 17:48, 27 October 2009 (UTC)
It is unreasonable to assume that there is a creature that evolved to be unable to roll over. Even Bender learned to get up off his back. -- kainaw 19:55, 27 October 2009 (UTC)
Unreasonable? DRosenbach (Talk | Contribs) 23:25, 27 October 2009 (UTC)
That source, which doesn't appear very reliable to my eye, doesn't state that the desert tortoise cannot right itself. It claims that desert tortoises cannot always right themselves, and it is suspiciously devoid of examples and references. Comet Tuttle (talk) 23:38, 27 October 2009 (UTC)
The point was that tortoises can get stuck depending on their shell shape and that it's unreasonable to state that actual reality is unreasonable. It's like saying that it's unreasonable that earthworms leave their flooded tunnels to get bloated and drown on rocks and eaten by birds and etc. when it happens all the time. It's like saying that it's unreasonable to assume that there is at least a single organism that cannot swim, because how would nature allow an animal to evolve into something so unfavorable - but I'm sure there are animals that drown (and that list, however short, might include caterpillars and some tortoises). DRosenbach (Talk | Contribs) 01:51, 28 October 2009 (UTC)
There is a huge difference between "having the possibility to get stuck upside down" and "being unable to roll over". If you rationalize that those two are equivalent, then we can pick anything that can happen sometimes and argue that it happens all the time. For example, why not argue that evolution sucks because humans always die in vehicles that humans invented and built. An obvious absurd argument - just as absurd as claiming that desert tortoises are unable to roll over because they sometimes get stuck upside down. -- kainaw 23:41, 28 October 2009 (UTC)

Mad Chicken Disease?[edit]

How do people know that the mad cow disease generally would not migrate to other non-ruminant species?

Chicken on modern farms are seldom observed by people. A chicken may only be examined once by people at hatching (sexing the chicken) and the second time in the slaughter house. If I have a little Old McDonald-styled farm, I would spend hours after hours watching my animals walk, run, eat. If a cow has become unable to walk, I would have noticed it. In contrast, a chicken farmer having a large industrial farm only check the chicken once a day. If a hen dies in the cage, he picks it up and send it to the rendering plant. I bet even if a chicken cannot work, it may not be observed before death.

In many countries, a dozen chicken are stuffed in a small battery cage. They are killed when they are about sex-weeks-old. I don't believe that you may notice any of their abnormal behaviors.

What may happen if I have a very large piece of land so I can grow one million free range chicken to their natural deaths? If I feed them with feeds made from mad cows and scrapie sheep and goats, would they show symptoms after a few years? -- Toytoy (talk) 17:37, 27 October 2009 (UTC)

The main way you would find out if your chickens are sick is to give a sample of them post-mortems, I think. I imagine that is being done. --Tango (talk) 17:43, 27 October 2009 (UTC)
Egg-laying hens in factory farms are kept about 1 1/2 years, and spongiform encephalopathy does eventually cause death, so perhaps it would be noticed by that route. Anyway, people eat an enormous amount of factory-farmed chicken -- including spinal cord tissue in the case of soups and stocks -- and there's no big prion-disease outbreak that needs explaining by a hypothesis of undiagnosed SE in chickens. --Sean 19:44, 27 October 2009 (UTC)
If chickens live at most 1.5 years for egg layers and much shorter time if intended for slaughter, then spongiform encephalopathy might not have time to manifest its behavioral effects. In humans, what is the incidence of mad cow/CJD and has it varied over the years? 24.12.190.7 (talk) 20:56, 27 October 2009 (UTC)

Many dog and cat food suppliers use livestock rendering as a main ingredient. Pet dog and cats may live for a decade or longer. They may eat the same brand of food for many years (prolonged consuming of the same ingredients). Many pet owners bring their pets to vets periodically. Despite the fact that extremely old dogs and cats are certainly too weak to walk, we do not see the global emergence of mad dog/cat diseases ... Does it imply that the species barriers are generally strong enough to stop a specific type of prion to migrate from this animal to that animal? -- Toytoy (talk) 03:13, 28 October 2009 (UTC)

We know that one of the major causes of the Mad Cow outbreak was that farmers were (perhaps unknowingly) causing highly processed left-over parts of cow carcasses back into protein that was been fed back to cows in their food supplements. The parts that were recycled were the things we don't eat - brain and spinal cord tissue - which is precisely the place where these prions are to be found in mad cows. Part of that was just sheer stupidity - it seems really obvious to me that cannibalism is going to cause diseases to spread - they should have thought of that. Worse still, cows are naturally herbivores - they don't have the digestive mechanisms to protect them from meat-born diseases.

Cats and dogs are both carnivorous - and (thankfully) we never feed them dog and cat parts - however indirectly. So neither of those major screw-ups that the cattle industry fell foul of could possibly affect cats and dogs. If a dog were to contract "mad dog" disease - it would eventually die - the body would probably be cremated or otherwise safely disposed of...there is simply no way for the disease to spread. You don't put dead dogs into dog food!

The case with chickens lies somewhat between those extremes. Chickens are naturally carnivorous - they eat all sorts of insects and worms and such - so they have a digestive system that's evolved to handle the problems that come up with eating meat. The less obvious question is whether left over chicken carcasses somehow get recycled into chicken feed. That was certainly possible in the past - and it seems quite possible that mad-chicken disease could have appeared in the past. But since we became acutely aware of the causes of the mad cow outbreak - I would assume that the feedstock chain is now likely to be very highly regulated to prevent dead chickens ending up inside chicken feed.

So, while it's not impossible for this to happen to chickens - I think the odds are considerably longer than they were with cows. SteveBaker (talk) 04:18, 28 October 2009 (UTC)

Most shockingly, this also can include dogs and cats. That's right, your pets could be cannibals. Fast Food Nation author Eric Schlosser writes, "Although leading American manufacturers promise never to put rendered pets into their pet food, it is still legal to do so. A Canadian company, Sanimal Inc., was putting 40,000 pounds of dead dogs and dead cats into its dog and cat food every week, until discontinuing the practice in June 2001.
However unlikely, some pet food companies might have used pet meat in their products. Since the majority of meat still has to come from fish and livestock, the concentration of any possible pet carcasses could have been small. And since pets eating "pet foods" might not re-enter the pet food chain when they die, their prions were unlikely to infect other pets. Take cats for example:
  1. cat carcasses --> dog food --> dogs are unlikely to be infected
  2. cat carcasses --> cat food diluted with other sources of meat --> cats are unlikely to be infected
  3. cat carcasses --> cat food --> infected cats --> buried or cremated --> cause no further infection
  4. cat carcasses --> cat food --> infected cats --> cat food --> more cats infected
Only #4 could result in the spreading of prion, I guess. -- Toytoy (talk) 01:49, 29 October 2009 (UTC)
It's definitely not implausible that pet food companies would use pet meat in their products if it were a cheap and available alternative to other meat, but that doesn't seem very likely. Where would the cat or dog meat be coming from? Pet owners don't sell their deceased pets' carcasses to meat vendors and I'm pretty sure animal shelters and pounds don't either. Rckrone (talk) 05:06, 29 October 2009 (UTC)
Were they from the puppy mills? -- Toytoy (talk) 09:13, 30 October 2009 (UTC)

code of silence[edit]

What is the code of silence called for doctors and for nurses? Biggerbannana (talk) 18:36, 27 October 2009 (UTC)

Doctor-patient confidentiality? --Tango (talk) 18:38, 27 October 2009 (UTC)
In the U.S.: HIPAA? -- Scray (talk) 18:55, 27 October 2009 (UTC)
From the Hippocratic Oath: "All that may come to my knowledge in the exercise of my profession or in daily commerce with men, which ought not to be spread abroad, I will keep secret and will never reveal." -- Scray (talk) 18:57, 27 October 2009 (UTC)
Sadly our article Hippocratic Oath is US Centric.. "Most Medicine schools administer some form of oath" as far as I know completely untrue in respect of every medical school in the UK for example, and I guess is unlikely outside the new world. I think Tango's answer is better --BozMo talk 19:07, 27 October 2009 (UTC)
My sister is a British medical student and I've never heard of any kind of oath for British doctors. --Tango (talk) 20:15, 27 October 2009 (UTC)
They used to, but they don't now. The principles are still there, but not the oath itself. - Jarry1250 [Humorous? Discuss.] 09:31, 28 October 2009 (UTC)
Used to in Hippocrates day in Greece maybe? Did they ever do this in the UK? It isn't mentioned in the history sections of the Royal Colleges websites as far as I can see. --BozMo talk 12:06, 28 October 2009 (UTC)
Found a slightly better reference in the letters section of the British Medical Journal suggesting the use of the HO in the UK ever is a myth: [14] --BozMo talk 12:11, 28 October 2009 (UTC)
Apparently it's also rare in Australia and NZ although a (small) majority of institutions do have a declaration of some sort [15] Nil Einne (talk) 21:11, 27 October 2009 (UTC)
For those (like me) who would check - it's been fixed to clarify that the statement applies to U.S. medical schools. -- Scray (talk) 11:20, 28 October 2009 (UTC)
coverup of malpractice? Gzuckier (talk) 19:59, 27 October 2009 (UTC)
I go with Gzuckier. Richard Avery (talk) 08:33, 28 October 2009 (UTC)

GVO pigmentation[edit]

What is GVO] pigmentation basically for when the primary color is green, orange, violet and secondary color is pink, yellow, blue. CYMK is use for printing and RGB is for light use.--209.129.85.4 (talk) 20:37, 27 October 2009 (UTC)

Secondary color is your friend :) --Dr Dima (talk) 21:36, 27 October 2009 (UTC)
The GVO color model is the reverse of the outdated subtractive RYB color model. Red Act (talk) 21:46, 27 October 2009 (UTC)

chemistry[edit]

what s the meaning of energetically favorable? —Preceding unsigned comment added by Farahghannam (talkcontribs) 20:37, 27 October 2009 (UTC)

Thermodynamics and chemistry aren't my strongest subjects, but I'm pretty sure I know the right answer. An energetically favorable reaction is one that decreases the Gibbs free energy of the system. Energetically favorable reactions can occur spontaneously, whereas an energetically unfavorable reaction can only occur if it is coupled to a second, energetically favorable reaction. It's basically a result of the second law of thermodynamics. Red Act (talk) 21:12, 27 October 2009 (UTC)
A reaction is energetically favourable if it has a negative ΔH - if it gives out more energy than it takes in. However, it may not be _kinetically_ favourable - it may take place so slowly that it effectively doesn't occur. See particularly Chemical kinetics#Free energy, and Chemical thermodynamics. Tevildo (talk) 21:13, 27 October 2009 (UTC)
Really? I'm rather weak on thermodynamics, but if I Google "energetically favorable", the search results are all about ΔG, not ΔH. Chemical kinetics#Free energy also mentions ΔG, not ΔH. I'm probably wrong, but I'd be curious to hear a brief explanation as to why. Red Act (talk) 22:57, 27 October 2009 (UTC)
Also, some reactions that occur are endothermic. Doesn't that violate the idea of negative ΔH always being what's energetically favorable? Red Act (talk) 23:47, 27 October 2009 (UTC)
It usually means that the reaction has a negative ΔG. The second law of thermodynamics tells us that the ΔG of the universe is always negative in the forward time (that is, free energy is always decreasing on a universal level). Any reaction that itself has a negative ΔG then will occur spontaneously; while a reaction that has a positive ΔG will require you to basically "steal" free energy from elsewhere to occur, so that the ΔG is negative in total for the universe. RedAct is basically right here. Enthalpy is only important because exothermic reactions decrease free energy for the universe (moving energy from a concentrated state inside a chemical bond to a diffuse state as heat); but the key measurement in determining energetic favorability is ALWAYS ΔG. --Jayron32 05:53, 28 October 2009 (UTC)

space expands faster than light[edit]

A widely talked about topic on the internet - in fact if you type 'space expands' into google, it expands it to the topic 'space expands faster than light'! This was mentioned when someone was talking about the Big Bang during The Universe Season 4 Episode 4, and piqued my interest. I kind of understand the possibility of the physics behind the concept, but I can't help thinking that once again, scientists have come up with contrived physics to explain the rapid expansion of space at the very start of the inflationary model. Where else do we see this happening? Certainly nowhere now except in the realms of science fiction... Sandman30s (talk) 21:46, 27 October 2009 (UTC)

The main problem with attempting to understand this concept is the idea that the edge of space is a thing. That thing cannot travel faster than the speed of light. The problem is regarding that there is a "thing" called the edge of space. A better way to conceptualize it is as the surface of a balloon. It has no edge. If you walk around on it, you can walk forever without reaching that thing called "the edge of the balloon". At the beginning of the big bang, the balloon was very tiny. Within seconds, it expanded to a huge size. The expansion rate was faster than the speed of light if you attempt to place the balloon in another space all together. For example, if you blow up a balloon real fast, you can say that two opposite sides of the balloon are moving away from each other at a certain speed - but that speed based on the distance measured between the sides of the balloon. What we call the speed of light is measured on the surface of the balloon - not inside the balloon. So, referring to an expansion speed has very little to do with the speed of light. If that all makes sense, then working out how the expanding size and distance between objects on the surface of the balloon does not affect the speed of light relative to the surface of the balloon even when the balloon expands. -- kainaw 21:53, 27 October 2009 (UTC)
I thought the 'thing' is a particle within space-time, not an edge of space itself? Anyway, I understand what you're trying to say... thanks. Sandman30s (talk) 22:35, 27 October 2009 (UTC)
You are right. Even if there were an edge of space, it could expand faster than the speed of light. Objects cannot move through space faster than light speed, that doesn't say anything at all about the metric expansion of space. --Tango (talk) 22:38, 27 October 2009 (UTC)
If you're generally asking, "does inflationary theory seem a little ad hoc to anyone else?" the answer is, emphatically, YES, it is, and even the scientists recognize this, BUT the data it gives fits REALLY quite well with experimental results, so most cosmologists think that there is a lot to it, EVEN IF it seems a bit ad hoc (at least, at this stage—it is still being worked on and is a topic of active research). It is, incidentally, not the first time in the history of physics that a somewhat ad hoc explanation to solve a numerical problem has turned out to be both true and much more profound than was initially realized—Max Planck's invention of the quanta was just such a thing (Planck thought of it as a heuristic, a mathematical "trick" to get the right answers, and it was some time before he or anyone else realized that it was a real, fundamental property of the universe, one that had massive implications for physics). --Mr.98 (talk) 22:15, 27 October 2009 (UTC)
Yes I was asking exactly that. What do you mean by 'experimental results'? Sandman30s (talk) 22:35, 27 October 2009 (UTC)
I meant observational results, actually, not experimental. Sorry. There is some detailed discussion of the Observational status in the theory article. And just to amplify one other thing—appealing to Planck isn't meant to say, "this is fundamental and must be true." I don't know—I'm no cosmologist, and even they aren't sure. It could be a Planck-type thing, where it ends up being true, or it could turn out to be a cosmological constant type thing, where the idea is fundamentally wrong. But I think the scientists are currently happy with inflation because the observational evidence works out well and it's one of those deals where it gives you good results with only ONE big arbitrary assumption (inflation), rather than the ton of tiny, arbitrary assumptions that other theories require. That's my understanding of it, anyway, as someone who does not understand the math in the slightest. --Mr.98 (talk) 22:40, 27 October 2009 (UTC)
I don't understand why you say that inflation is ad hoc. All you need for inflation is general relativity and quantum field theory and a scalar field with some fairly unremarkable properties. Anyway, this isn't related to the original question. Inflation or no inflation, space doesn't expand faster than light, and you should be suspicious of the overall accuracy of any book or TV program that claims that it does. -- BenRG (talk) 22:55, 27 October 2009 (UTC)
I'd like to know why you think that. AFAIK "The Universe" makes use of some eminent cosmologists from top universities. The one who made that statement was Alexei Filippenko, American astrophysicist and professor of astronomy at the University of California, Berkeley. Sandman30s (talk) 10:41, 28 October 2009 (UTC)
I watched the episode (the big bang part of it, at any rate). Here's a transcript of what Alex Filippenko said:
"The 'bang' of the big bang is a time in the early universe known as inflation. The size of the universe went "whoosh"—it just expanded so quickly, doubled in size many times over a very very short interval of time. This incredible inflationary exponential state of expansion lasted only a tiny fraction of a second. A millionth of a millionth of a millionth of a millionth of a millionth of a millionth of a second."
Then there's some voiceover narration, and then:
"In fact, it expanded so quickly that it was faster than the speed of light. That's okay—that doesn't violate Einstein's theory of relativity. Space itself can expand faster than the speed of light. No particle can travel through space faster than the speed of light, but space itself can and does expand faster than the speed of light."
So let's look at inflation. In the diagram on the right, the curve to the right of the point marked "reheating" is a square-root curve; this is the expansion predicted by general relativity when the mass of the universe is mostly in the form of particles moving at relativistic speeds. The dotted line is the extrapolation of the square-root curve back to the point where it hits zero, which is called the "big bang singularity". That was the traditional model. In the inflationary model the universe is only dominated by relativistic particles back to the time marked "reheating", and before that it's dominated by a scalar field; according to general relativity, this gives an exponential expansion curve instead of a square root. This is not the only such transition in the universe's history; around 100,000 years later there was a transition from relativistic particles (t1/2 curve) to nonrelativistic particles (t2/3 curve) and right now we seem to be in the middle of a transition back to scalar field dominance (the famed accelerating expansion). Parts of this model may turn out to be wrong, but at any rate that's the present model.
The first thing you notice about the inflationary expansion is that it's much slower than the traditional big bang expansion (given by the dotted line). It is fast in absolute terms—in order for the inflationary epoch to solve the problems it was introduced to solve, the scale factor has to increase by a factor of at least 1025 or so, and this happens in a tiny fraction of a second. I can't remember how long it takes, but let's take Filippenko at face value and say that it's 10−36 second. A factor of 1025 in 10−36 second is fast, no question, but the traditional big bang expands by a factor of infinity (from zero to a nonzero size) in an even smaller time (less than a hundredth as long). You could argue that people are technically telling the truth when they say inflation is fast, but when it's part of a story that goes "inflation was introduced into the traditional big bang model, and inflation is really fast", no one is going to come away thinking "I bet the traditional big bang expansion that inflation replaced was even faster". I don't know where this fast-inflation narrative came from; it seems completely arbitrary.
Additionally, no one knows how long inflation lasted. The fact that you need an expansion factor of at least 1025 gives a lower bound, but the simplest inflationary models last enormously longer than that; they have no problem satisfying that lower bound, in other words. So anyone who says that inflation lasted for 10−n second, for any n, is making it up. Nobody knows. A lot of people understandably have the idea that if inflation ended 10−k seconds after the big bang then it must have lasted 10−k seconds or less, but as you can see from the diagram it doesn't work that way. Times quoted as "since the big bang" are measured from the traditional big bang singularity (by necessity, since the duration of inflation isn't known), so inflation starts "before the big bang".
I'm not sure what to make of the claims of faster-than-light expansion. There are two versions of this, one claiming that space always expands faster than light, and the other claiming that it only expanded faster than light during inflation. The former is presumably referring to comoving recession velocities, which can exceed c, but this really has nothing to do with faster than light motion, which involves a different kind of velocity. The best I can come up with for the latter is that it's referring to the presence of a cosmological event horizon during inflation; effectively the universe is partitioned into regions that can't communicate until after inflation ends. But that's not at all what your typical layperson will imagine as "space expanding faster than light". They will have in mind some kind of absolute background like Newton's absolute space, and space as a separate substance moving through that with a certain speed, and that bears no resemblance to reality.
Why did Alex Filippenko say those things? I don't know. Maybe I'll ask him. -- BenRG (talk) 15:18, 28 October 2009 (UTC)
Many thanks for the in-depth explanation. If the prof replies to your question, I'd love to know his answer. If that comment was tongue-in-cheek, maybe you should try to email him anyway! Keep me posted! Sandman30s (talk) 21:11, 28 October 2009 (UTC)
Er, I'm pretty sure the OP has made it clear that the ad hoc question is exactly what was meant by the original question (title be damned). What I mean by "ad hoc" is that the inflationary epoch was created to solve a problem of the previous existing theory. It is a "patch" to fix a cosmological model that otherwise doesn't work. You have a little period of really abnormal behavior, stuff that doesn't generally happen, and then things go back to normal again. (Why did the abnormal period start? Why did it stop? Unclear.) Yes, if you set all the fields up right, it works fine, but that's tautological (if you do what the theory says, the theory works). Historically it was conceived of as a "patch", and I have read scientists today who talk about it as a "patch". Still, as patches go, it works pretty well, in terms of observational results. The question is: is it physically real, or is it a mathematical fix? That's what I'm trying to get at up there. --Mr.98 (talk) 01:39, 28 October 2009 (UTC)
I just don't understand why you're singling out the inflationary epoch. The post-inflationary state that you describe as normal was pretty weird. The forces were unified, quarks were unconfined, there was some CP-violating process going on that resulted in the present-day dominance of matter over antimatter, etc. The accelerating expansion in the inflationary epoch doesn't happen by fiat, but because that's what general relativity predicts for a universe dominated by a scalar field. The transition to radiation-dominated expansion happens when the scalar field reaches a local minimum of its potential. The model was constructed to fit existing experimental data, but so was Newtonian mechanics. It also made predictions that were confirmed later (scale invariance of the CMB power spectrum). It's based entirely on well-tested theories, GR and QFT, and all it needs is a new particle. All of the alternatives to inflation require radical new physics. It's hard to test, but that's unavoidable—that era is hidden from us by 400,000 years of plasma. -- BenRG (talk) 16:02, 28 October 2009 (UTC)
I'm singling it out because I've seen others single it out, and because inflation was a rather later-comer to the Big Bang question. I'm not, as I indicated above, claiming that inflation is wrong or weird. But it was, at the time, perceived to be a hack, and there are still some of those who consider it to be one. I'm personally fine with that. Obviously if you have internalized a theory, it looks natural and non-hackish. --Mr.98 (talk) 13:28, 29 October 2009 (UTC)

What sort of medical data is stochastic?[edit]

I am not asking for medical advice here. I am asking about measurements.

I'm trying to learn what sort of time series data would be measured by a medical monitoring device that is stochastic or has the appearance of randomness?

The only things I can think of so far are:

  • EEG data measured from patients with epilepsy
  • Insulin levels self-measured by diabetics

Is there anything else, especially medical time series data that defies prediction efforts? =Axlq 21:49, 27 October 2009 (UTC)

Hmm...even after checking out the stochastic article linked above, I can't make a real grasp of it in order to give you any examples -- perhaps a few examples with the reasoning behind them would help those very familiar with medicine/science yet not so familiar with stochacism (if that's the noun form). DRosenbach (Talk | Contribs) 23:29, 27 October 2009 (UTC)
According to SOED: stochasticity noun, the property of being stochastic Mitch Ames (talk) 00:03, 28 October 2009 (UTC)
"Stochastic" is what statisticians say so they sound clever. It just means "random". --Tango (talk) 00:33, 28 October 2009 (UTC)
My sense is that in biostatistics (and some other disciplines), "stochastic" is used to emphasize more discretely random data (e.g. a random walk), as contrasted with more continuously random data. I'd agree that there's no bright line. -- Scray (talk) 01:44, 28 October 2009 (UTC)
Pretty much all continuous measurements can modelled as stochastic. Even if there is no randomness is the quantity itself there will be random error in the measurement. --Tango (talk) 00:33, 28 October 2009 (UTC)
Stochasticity is a very old but (no pun intended) inexhaustible subject of research in neuroscience. Neurons in the central nervous system do not respond in an exactly repeatable way to a repeated stimulus; some randomness is always present. In absence of a stimulus, spontaneous activity in neural networks - in vivo, in vitro, or simulated - is very often stochastic in its nature. --Dr Dima (talk) 00:38, 28 October 2009 (UTC)
I don't think that insulin levels are routinely checked by diabetics. Perhaps you mean blood glucose monitoring. Other measurements to consider: urine sodium (fluctuates over a wide range), pulse oximetry (pulse and imputed oxygen saturation), pleural pressure variation measured in a chest tube canister, and temperature. The list is almost endless. -- Scray (talk) 01:00, 28 October 2009 (UTC)
From what I can put together after all of these explanations and exemplifications, I wonder if there are any measurements that are not stochastic -- I mean, height is probably not stochastic, but weight (to the gram, let's say) would be stochastic, wouldn't it? My point is that I didn't understand your question before, but I don't really understand it now, but for a different reason (because examples seem to abound to the extent that such a question is silly). Anyway, maybe I just don't understand it at all -- that's certainly a possibility. DRosenbach (Talk | Contribs) 01:36, 28 October 2009 (UTC)
(ec) Cost of service can be modeled stochastically. That is the basis of the entire health insurance industry - a particular individual's cost for healthcare is unknown (though can be modeled using prior knowledge about their health, and statistically analyzed as a member of a larger population). In truth, every data series can be modeled stochastically - some data is simply more "random" because the controlling parameters are less well-known. Nimur (talk) 20:28, 28 October 2009 (UTC)

OK, thanks for all the comments, but I think the answers are focusing more on semantics than the intent of my question. Suppose I have an algorithm to which I can feed a random-walk time-series signal, and that algorithm does something predictive with the input signal (for example, say it identifies data points that it predicts will be the beginnings and ends of trends). Now, I want to apply that algorithm to the field of medical monitoring devices. I'm looking for appropriate medical measurements that might benefit from this predictive algorithm. All I could think of were EEG measurements and (thanks for the correction) blood glucose measurements (which exhibit randomness because they are typically done at irregular times). =Axlq 18:23, 29 October 2009 (UTC)

The heartbeat has sometimes been used for this -- there is a degree of stochasticity in the series of intervals between beats. There are actually lots of variables showing a mix of structure and randomness -- eye blinks and eye movements are other examples. Looie496 (talk) 18:39, 29 October 2009 (UTC)

Francium in water[edit]

Where's a video of francium reacting with water? I looked on YouTube, but I couldn't find any. --70.245.188.147 (talk) 23:30, 27 October 2009 (UTC)

Hardly surprising. According to our francium article, The largest amount ever collected of any isotope was a cluster of about 10,000 atoms (of francium-210) created as an ultracold gas at Stony Brook in 1997. Wouldn't make much of a video. --Trovatore (talk) 23:32, 27 October 2009 (UTC)
But given the trend with the reactivities of other alkali metals, isn't francium reactive enough that even a mere 10000 atoms would cause a large explosion? --70.245.188.147 (talk) 23:45, 27 October 2009 (UTC)
Large? No. 10000 atoms is almost nothing. That's only about 1.6x10-20 moles. And with anything that extremely hard to gather, they aren't likely to waste any of it making videos of it reacting with water. Red Act (talk) 23:57, 27 October 2009 (UTC)
Even if you converted the atoms completely into energy (E=mc^2 and all that) you would only get 0.0003 joules. That's not much of an explosion. --Tango (talk) 00:27, 28 October 2009 (UTC)
Fun fact: a single fissioning uranium atom will plink a grain of sand about an inch off the ground releases enough energy to make a grain of sand visibly jump, so 10,000 of those might be fun to watch. --Sean 02:27, 28 October 2009 (UTC)
Um, really? Let's do a quick estimate: A uranium atom weighs 235/(6e23) grams -- call that 4e-27 kg as we don't need much accuracy. If you could completely convert it to energy (which is off by three orders of magnitude) that would be about 4e-10 J. A grain of sand weighs at least a microgram, 1e-9 kg; to raise that one meter would take 1e-8 J, so to raise it one inch would take about 3e-10 J. So if fissioning converted all the mass to energy, you'd be in the ballpark. But as I say, that's three orders off (the Hiroshima bomb fissioned about 1kg of U-235, and its yield was the equivalent of about 1g of mass). --Trovatore (talk) 08:05, 28 October 2009 (UTC)
Maybe a single pair of annihilating uranium (anti)atoms? Still a fun fact. Someguy1221 (talk) 08:20, 28 October 2009 (UTC)
Fun facts are sometimes more fun than factual.
Dilbert: Studies show that numbers you make up are just as useful as the real ones.
Audience member: How many studies show that?
Dilbert: 87.
--Trovatore (talk) 08:23, 28 October 2009 (UTC)
Here's another point to consider, in case anyone wants to argue for nanogram "grains of sand" (I think those'd be more like dust motes, but whatever). The word "plink" suggests that the visible particle is going to be propelled into the air as a direct consequence of the fission, not by some complicated micro-winch powered by the fission. For that, as in a previous discussion of reaction drives in space, the limiting factor is not energy at all, but rather momentum. The momenta of the reaction products are very small; even if the upward-bound reaction products were all absorbed inelastically by the mote of sand, almost all of the energy would simply go into heating the mote, not lifting it. Imagine shooting a bullet into a boulder — as small as the velocity would be that corresponds to the bullet's energy, the boulder isn't going to move anywhere near that fast; almost all the energy would be simply dissipated.
OTOH if the sand mote is perfectly rigid and you get an elastic collision, then the mote will move slightly faster than it would from the inelastic absorption. But still not enough to notice — almost all the energy will be carried away by the reflected reaction products, and almost none will go into the sand mote. --Trovatore (talk) 08:48, 28 October 2009 (UTC)
What I have heard said, by physicists at various times, is that on average, U-235 fissions with about 200 MeV of energy, which is about as much energy to move a speck of dust noticeably. Notice that nobody said that fission itself actually moved it—you'd have to get all 200 MeV pointed in the right direction as kinetic energy, which isn't actually going to happen in real life. The nice thing about the speck-of-dust analogy is that then you (the teacher) say, "that's kind of a lot for ONE atom, no? Now if you increase that to 1kg of U-235, that's a LOT of dust—a whole city's worth!" and so forth. --Mr.98 (talk) 13:14, 28 October 2009 (UTC)
Here is my source for the claim. It actually says it will make the grain "visibly jump", which I misremembered as "jump an inch". I thought it was obvious that I meant "the energy of one fission event", rather than a proposed Atomic Sand Plinker, but have amended my statement just the same. --Sean 13:27, 28 October 2009 (UTC)
Well, when I write something, its meaning is almost always obvious to me :-). You can see, I hope, that a naive reader might suppose that if you had a plate made of U-235, and sprinkled sand on it, you'd every now and then see a sand grain move when a uranium atom captured an ambient thermal neutron. And that I think is completely wrong, for the reason I explained. --Trovatore (talk) 20:09, 28 October 2009 (UTC)
Re: obviousness, it reminds me of the Ignobel Prize committee awarding the prize for the water-memory homeopathy guy who "demonstrated to his own satisfaction that water retains information that can be transmitted over the Internet". :) --Sean 14:37, 29 October 2009 (UTC)
Yeah, but this isn't fission here. It's just oxidation in water; alkali metals tend to do so in a highly exothermic manner. The deal with Francium is, while in theory, it should be even more reactive than Cesium (to date, the most reactive observed metal), there has never existed in one place anything more than an exceedingly microscopic number of atoms. Even if every atom of francium that has ever existed on earth were dropped in water at the same time, it would likely not generate enough energy to fizzle. --Jayron32 05:42, 28 October 2009 (UTC)
The other problem with this idea is that Francium isn't stable. The most stable isotope has a half-life of just 22 minutes - so if you make your 10,000 atoms of the stuff - then take a break for coffee - you'll only have 5,000 atoms left when you get back! SteveBaker (talk) 12:17, 28 October 2009 (UTC)
In that case, put it in your accelerator and speed it up to near the speed of light before your coffee break so time for it will go more slowly as compared to your frame of reference. :) 20.137.18.50 (talk) 14:56, 28 October 2009 (UTC)
... but then its kinetic energy (+ other Taylor expansion terms) will swamp its rest-mass energy by many orders of magnitude, and any chemical reaction with water will be totally insignificant. Dbfirs 19:59, 28 October 2009 (UTC)


October 28[edit]

Food safety question[edit]

So I bought this delicious jar of imported sun-dried tomatoes in olive oil. Yum. Trouble is I don't know where to keep it now that the jar is open. If I put in the fridge, the olive oil congeals into an unappetizing, solid yellow cylinder. If I keep it the cupboard I worry about bacteria breeding in the oil. What's a Fat Man to do?--The Fat Man Who Never Came Back (talk) 00:03, 28 October 2009 (UTC)

Does it not have storage instructions on the label? --Tango (talk) 00:11, 28 October 2009 (UTC)
No (it's from Turkey; they don't have food safety over there).--The Fat Man Who Never Came Back (talk) 00:13, 28 October 2009 (UTC)
What's a fat man to do? Eat them, of course! Seriously, if you don't let them get damp, they should keep for at least a week out of the fridge -- there's basically no form of life that can grow without water. Looie496 (talk) 00:19, 28 October 2009 (UTC)
Oil and water don't mix very well. Safety isn't a big issue, and you may want to read water activity as an explanation of why bacteria breeding is not a big deal for things packed in oil. Having the oil go rancid from exposure to air is more of a quality concern than a safety issue. If there isn't any water in it, there won't be any bacteria growing either. There are still bacteria there, but most are spore-formers and other inactive forms. SDY (talk) 00:20, 28 October 2009 (UTC)
The stop them getting damp, you should make sure they are all completely covered by the oil. If they aren't, add more oil. --Tango (talk) 00:21, 28 October 2009 (UTC)
They were sun dried to remove the water then put in oil to keep them away from bacteria. This method was invented before fridges were even thought of. Put them where you like and they'll last for months. 08:32, 28 October 2009 (UTC) —Preceding unsigned comment added by Richard Avery (talkcontribs)
That is a good general point - a lot of foods were invented as ways of preserving fruits and vegetables over winter (jams/jellys, chutneys, pickles, etc.). They will last months without us doing anything further to preserve them. --Tango (talk) 15:49, 28 October 2009 (UTC)
Thank you, your answers make sense. However, perhaps what provoked my original question was a chef I respect once warning us to be very careful when making homemade garlic-infused or herb-infused oils, as improper storage or handling can result in botulism contamination. Why would botulism bacteria grow, even if the product were submersed in oil? Is this because of the water content of the garlic or herbs?--The Fat Man Who Never Came Back (talk) 00:02, 29 October 2009 (UTC)
I can't say anything about the safety, but I will note that submersion in fat (Confit) has a long history as a food preservation technique. -- 128.104.112.149 (talk) 22:31, 29 October 2009 (UTC)

Voice differences between blondes and brunettes[edit]

After comparing some samples I have a feeling, that brunette girls might have a higher pitch than blondes, who feature a bit deeper voice as compared to brunettes. Are there actually any differences in general (including red-haired girls)? —Preceding unsigned comment added by 217.25.31.177 (talk) 00:51, 28 October 2009 (UTC)

Perform a double blind randomized clinical trial and let us know. DRosenbach (Talk | Contribs) 01:30, 28 October 2009 (UTC)
That's going to be a tricky experiment for a non-professional to arrange. Persuading a statistically reasonable number of blonde and brunette girls (20? 50?), whom you've never met before (they can't be people you might recognise), to talk to you while you're blindfolded without telling them why (that's important - so as not to bias the results)...not so easy! If you're going to do it - make sure you control for age - and if possible, find some people who dyed their hair so you can figure out whether that's significant. I would be somewhat surprised if this were true - but it's perfectly possible and would make a great science project. But the real issue here is: "Can we find any pre-existing evidence?" - I couldn't find anything with Google or Wikipedia searches. SteveBaker (talk) 02:07, 28 October 2009 (UTC)
Well, let's consider natural color with the age between 18 and 30. However this depends on country, it should have a significant amount of girls with both hair colors to conduct trial. 94.20.22.147 (talk) 10:16, 28 October 2009 (UTC)
You would still have to correct for age. There is quite a lot of research on female voices, dominance and attractiveness but pitch lowers with age continually 18 to 30. The dominance thing matters too though. Both men and women naturally raise the pitch of their voice when there are people whose dominance they accept present and lower the pitch when asserting. You get taught this kind of thing in body language courses. To the extent that hair colour affects confidence affects assertiveness it is even possible hair dye will change pitch. --BozMo talk 11:55, 28 October 2009 (UTC)
You could go to Amazon Mechanical Turk and request people to enter their age, hair color, and a short audio clip of them saying a set phrase, without telling them what this is all about, and then listen to the recordings without letting yourself see the hair color they entered. Or to homogenize data, you could specify that only females in a certain age range are eligible for this HIT (Human Intensive Task). You'd have to give some small amount of money for each piece of data, as 10-15 cents is common.20.137.18.50 (talk) 15:02, 28 October 2009 (UTC)
I'm not aware of any experimental data that addresses this question. However, mechanistically speaking, I would suggest it is unlikely. There are some genes known that result in hair colour differences and none of them are known to be involved in the formation or function of vocal folds. Of course, it could simply be that no-one has studied them closely enough yet. But the idea that hair colour being correlated with other characteristics is not unheard of, for example, there is a known mechanistic relationship between hair colour and pain sensitivity (e.g. PMID 15731586, PMID 12663858 ). Rockpocket 18:18, 28 October 2009 (UTC)
Indeed - but it doesn't have to be genetic. If (for example) the "blondes have more fun" and "gentlemen prefer blondes" memes were influential - perhaps blonde females might subconsciously adjust their vocal range to somehow try to seem more child-like or something. I'm certainly not saying that this happens - or that this is even a viable mechanism - but it's perfectly possible that something learned in childhood or teen years that is culturally related to hair color has something to do with it - and hence, genes need not be involved. SteveBaker (talk) 22:45, 28 October 2009 (UTC)
No one yet has raised the issue of the correlation between age and dying dyeing one's hair blonde. My purely WP:OR wild guess as to what might be going on is: A very young (and hence high-voiced) dark-haired woman doesn't have much disposable income, and gets plenty of attention from men without much effort, so she isn't all that motivated to spend the time and money to get her hair dyed blonde. A somewhat older (and hence lower-voiced) dark haired woman has more disposable income, and is more motivated about trying to hold onto her fading youthful looks in whatever ways she can, so she's more likely to dye her hair blonde. Genetics has nothing to do with it. Red Act (talk) 23:19, 28 October 2009 (UTC)
Um, dyeing rather than dying, unless there's some deeper thing going on here.... --Trovatore (talk) 23:25, 28 October 2009 (UTC)
Whoops, that's one the spell-checker can't catch. Red Act (talk) 23:33, 28 October 2009 (UTC)
Steve's suggestion is certainly possible, however vocal range is mechanistically associated with structural characteristics of the vocal folds. One has the ability (subconsciously or not) to affect a change in vocal pitch, but only with a limited range. Beyond that range, one's pitch is limited by biophysics and anatomy. A serious study into this would be able to distinguish between affected and biological differences in tone. The problem with social or environmental effectors is that they would be near impossible to control (given the propensity and popularity of hair dye these days). Finding a genetic effect would probably be the only way to show a statistically valid difference, since you can dye your hair as often as you like, but your genotype always remains the same.
As for the issue of age, any statistically robust study would use age-matching (did the OP do likewise in his or her survey?) Rockpocket 23:49, 28 October 2009 (UTC)

plz help fft plots[edit]

Moved to Computing desk - Falconusp t c

Hard drive for archiving video?[edit]

I have some videos I want to keep long term,indeed beyond my lifespan.If I stored them on a hard drive recorder,computer or plug in hard drive and did not use the drive again, how long would it be before the video or other data started to degrade irretrievably, compared to DVD or Blu-Ray disks stored in the dark and not played for example?80.2.204.58 (talk) 10:58, 28 October 2009 (UTC)

After you are dead the chances are that those cleaning up after you will through them away. Your best chance would be through massive replication on many servers. Constant maintenance will be required to keep the file in a format that can be used, as your codec may not be down loadable in a few decades time. I would recommend a free open source codec like ogg. Store the software to play it and its source along with your video too. Graeme Bartlett (talk) 11:40, 28 October 2009 (UTC)
There really isn't a medium for stuff like that which won't either physically degrade or become so obsolete that it won't work anymore. For example: When I was a kid, about 50 years ago, my parents took massive amounts of footage of my sister and me using "Super-8" cine cameras. Now, to play back Super-8 is almost impossible - even if the film stock is still in good shape (which it probably isn't). There are a few specialist places that'll convert it to something else - but they are horribly expensive. Give it another 50 years and it'll require someone in a museum to play back Super-8 (consider the difficulties in replaying a 1900's piano roll recording using one of the myriad of piano roll "formats" - or recovering sound from a 1900's Edison drum recorder or a 1920's wire recorder). I have software that I stored on my Atari-ST computer - there is no way for me to easily get that back - even if the floppy disks have survived.
In my opinion, the best way is to keep it on the Internet on some large public server. If the company that runs it is large enough, they'll make backups and copy your data from old, obsolete hard drives onto whatever futuristic technology comes next as a natural part of their upgrade cycle. That doesn't protect you from the file formats becoming so obsolete that they won't play anymore - but it's probably your best bet at keeping your videos intact. Put them on YouTube and on WikiCommons and on as many other similar sites as possible - one might hope that when they eventually decide to change file formats, they'd go through their existing video collections and convert them to the new format. You are unlikely to be able to do that and keep them private through - and of course it's not a cast-iron guarantee - look what happened to GeoCities this week - but at least the folks on the Internet Archive will keep copies as long as they stay in business.
But who knows? The thousands of people who recorded their own performances on piano rolls probably thought their formats would last forever too. It's likely that only the very best "most important" piano roll performances ever been transcribed onto vinyl disks and from there onto CD's and from there onto MP3's. Who knows how many of those will fall by the wayside whenever the next great thing comes along?
The migration of things like music and video into bits and bytes probably gives them the best chance at longevity because bits and bytes can be copied endlessly without degradation - but that was true for piano rolls too - and standards shift and cultural change has not done so well for them.
SteveBaker (talk) 12:08, 28 October 2009 (UTC)
And, in fact, because they are on paper, the piano rolls will probably last a few hundred years, providing they are stored well. The same cannot necessarily be said for magnetic media at all. The problem with the piano rolls is the player; the problem with magnetic media (or plastic media) is actual physical degradation over time, plus the player.
You would think there would be a market for long-term data preservation—e.g. a company that stored your data (under whatever terms you wanted), and then promised to not only make lots of copies of it in different locations, but to every five years or so transfer it to whatever the new storage medium format is at the time, etc.
Obviously it would only last as long as the company did, though, and I guess that's actually probably where the worst odds lie. --Mr.98 (talk) 13:07, 28 October 2009 (UTC)

Maybe a good bet would be to persuade the BBC that it was worth archiving?80.2.195.144 (talk) 14:01, 28 October 2009 (UTC)

I would think piano rolls could be recovered easily enough. Just run them through an optical scanner. The rest is a routine software project. If no one has yet done this, could be a business opportunity. --Trovatore (talk) 18:56, 28 October 2009 (UTC)
Sure, for someone with a lot of technical background - some highly specialised software, a large-format scanner, etc. But that's what I'm saying - after 100 years, it takes the skills of a relative expert to recover the information. And it's a lot easier with something physical like a piano roll - there is a simple 1:1 correspondence between the holes and slots and the sounds. Try that 100 years after the event with a poorly-documented, horribly complicated binary video file - possibly burdened with DRM encryption and who-knows-what else. It's not that it can't be done - it's that it takes the services of a proper antique media restoration service - and that's never going to be cheap. In 10 years time, you can probably still replay it on any PC you have at home. In 20 years, you'll have to ask around friends - look for old computer clubs - maybe find an emulator. In 40 years, it'll be impossible to run that old software and you'll probably have to pay some company a pile of money to get it back for you. In 100 years - you'll need the services of a museum-grade expert - who'll probably only consider doing it for things that have historical merit and as a part of some bigger on-going research activity.
That's why putting it on the Internet with something like YouTube would be best - assuming the company you choose doesn't go bust (and you could place it with half a dozen companies to ensure against that) - they'll upgrade their hardware as needed - migrating existing files over onto the new solid-state drives - or the holographic laser storage unit - or whatever the next trendy mass-storage device turns out to be. They'll keep backups as needed - and if the format they use for storing video ever has to be superceded, you know that they won't just ditch all of the existing material - they'll write conversion software and convert to whatever new format is trendy. This incremental tracking of the technology is what will keep your files usable. However, this is still not a cast-iron guarantee - look what happened to GeoCities yesterday. SteveBaker (talk) 22:38, 28 October 2009 (UTC)

maximum frequency infinite or not?[edit]

Is maximum electromagnetic radiation frequency almost infinite (which means nearly an zero wavelength) or no limit with an infinite frequency and zero wavelength? Biggerbannana (talk) 12:43, 28 October 2009 (UTC)

The frequency must be finite. The electromagnetic radiation must consist of at least one photon, and the energy of a single photon is E=hν, where ν is the photon's frequency, and h is the Planck constant. So the frequency must be finite, because an infinitely large frequency would require an infinite amount of energy, which doesn't exist. Red Act (talk) 13:20, 28 October 2009 (UTC)
That's a cheap out, and not even certain - how do you know that an infinite amount of energy does not exist? A more interesting question would be if there is a theoretical limit to the wavelength of EM radiation. What if it becomes smaller than Planck length? What if the photon has enough energy to become a black hole? BTW, Biggerbannana, classical thermodynamics predicts unlimited energy photons from cavity resonators (or black bodies) - see Ultraviolet catastrophe. Einstein and Planck saved us from that... --Stephan Schulz (talk) 13:30, 28 October 2009 (UTC)
We know energy is not infinite because the universe behaves as though it has a finite amount of energy (or more properly energy/matter; since they are one in the same). If the universe had an infinite amount of energy, then it would have an infinite amount of free energy as well, and there would be no thermodynamic drive for work to occur. A photon could not contain an infinite amount of energy because then it would contain all the energy of the universe and then there wouldn't be any energy left for, you know, the rest of the universe. I suppose one could consider a pre-big-bang (or an instant-of-big-bang) something like an "all of the energy in the universe" particle, but in the current universe, once all the laws have settled out, such an idea makes little sense. --Jayron32 13:48, 28 October 2009 (UTC)
Sorry, I don't see how your claim follows. Our current models favor, but do not require an (unbounded) finite universe, but are compatible with a flat, open universe. In that case, total energy is almost certainly infinite (or average energy density would be zero). Yes, energy in the observable universe is certainly finite, but that's a different question. Anyways, the limits I suggested as possible are much stricter than "all the energy in the universe". Unless I miscalculated somewhere, a photon at Planck length would have 12.415GJ, the mass equivalence of 0.13 microgram. --Stephan Schulz (talk) 14:58, 28 October 2009 (UTC)
If a photon had an infinite amount of energy, it would form an infinitely large black hole. If such a thing existed in the visible universe, it would be pretty darn hard to miss. So a ridiculously pedantic answer might be more along the lines of "There is strong evidence that there does not exist a single photon with an infinite frequency, anywhere within the visible universe."
However, the OP looks to me like a straight-forward question, looking for a straight-forward answer. Bringing up infinite-energy black holes in response is a little like getting a question like "What insect is this?", and replying that there's a possibility that the photo might be of a robotic spy, carefully constructed to look like a insect by ancient inhabitants of a planet in orbit around Betelgeuse. That might be true, and that possibility might be fun to think about, but that's not really the kind of answer being requested. Red Act (talk) 15:35, 28 October 2009 (UTC)
I think we disagree about the interpretation of the question. It's not about an infinite energy photon - the question is if there is an upper limit to the energy of a photon. Each number in N is finite, but there is no upper limit. Your reply (there is only a finite amount of energy in the universe (debatable, but granted for this discussion), therefore the energy a photon can have is bound by this limit) is obviously correct, but it avoids the issue if there is a limit dictated by the laws of physics, not by the boundary conditions. If there is such a boundary, it could be much stricter than the one give by your argument. --Stephan Schulz (talk) 15:46, 28 October 2009 (UTC)
Any upper limit to the energy of a photon would break relativity, as there is a point of reference from which any given photon has any given amount of energy. — DanielLC 20:05, 30 October 2009 (UTC)

The article Electromagnetic spectrum says various things about the upper frequency limit: Its wavelength is thought to be in the vicinity of the Plank length; in principle there is no limit; it is 2.4x1023 Hz (1 GeV gamma rays); but 2.9x1027 Hz has been detected from astrophysical sources. See GZK limit for a theoretical upper limit to cosmic ray energies. However it seems not every cosmic ray got the memo. Cuddlyable3 (talk) 17:07, 28 October 2009 (UTC)

Also, of course, cosmic rays aren't, and are not part of the EM spectrum. --Stephan Schulz (talk) 17:16, 28 October 2009 (UTC)

Actually what I was hinting at/searching for was the highest possible frequency/shortest possible wavelength to determine from a lay point of view what particle it might be associated with or become. At some point it seems clear that particles such as a neutron for instance must express electromagnetic resonance of a particular frequency or wavelength short of infinite frequency. Is it a gamma ray or a cosmic ray or what. (Excuse my rather boring lack of understanding.) Biggerbannana (talk) 17:15, 28 October 2009 (UTC)

You may find de Broglie wavelength interesting. --Tango (talk) 21:50, 28 October 2009 (UTC)

Maillard Reaction in Pasta Production[edit]

I am trying to find more information on Pasta Production, how would you prevent the product form changing colour. The article “Maillard Reaction” did not provide enough information. Regards, Jeanette —Preceding unsigned comment added by Jeanette de Lange (talkcontribs) 14:27, 28 October 2009 (UTC)

When I make pasta, I don't subject it to heating and therefore there will be no Maillard Reaction. Why do you think pasta production involves the reaction? --Phil Holmes (talk) 14:36, 28 October 2009 (UTC)
(edit conflict with Phil Holmes) How are you making your pasta? Fresh pasta is basically uncooked dough, and dried pasta is just dried, uncooked dough. There is nothing in the making of pasta that is hot enough to cause the Maillard Reaction to occur. It's all done at room temperature. Likewise, most pasta is cooked via boiling, and since the Maillard Reaction occurs at much hotter temperatures than the 212F/100C that water boils at, it is does not occur during the cooking process either. The ONLY applications I could see where the Maillard Reaction may occur is in applications like Fried ravioli or in some baked dishes, like lasagna, but generally only around the edges of the lasagna caserole, where the noodles are exposed and out of the sauce. --Jayron32 14:40, 28 October 2009 (UTC)

Why ozone is over antartica?[edit]

We know that there is ozone hole(depletion of ozone) over antartica. And that becoz of CFCs emmitted by us.. my question is that "when we are polluting air above us then why ozone layer above Antartica is effected". I apologise for my english.—Preceding unsigned comment added by 117.200.54.99 (talkcontribs)

Does our article on Ozone depletion help? In short, it's because the chemical reactions that break down ozone are very much more efficient if polar stratospheric clouds catalyze the reaction. This type of clouds forms only at very cold temperatures. --Stephan Schulz (talk) 15:21, 28 October 2009 (UTC)
That's horrible.--Gilisa (talk) 17:09, 28 October 2009 (UTC)
What's horrible? His description was accurate and hard to put simpler. Regards, --—Cyclonenim | Chat  18:03, 28 October 2009 (UTC)
Not his description, the idea that as the hole in the ozone layer is grown, earth get warmer, icebergs melt, and there is no chance for the ozone layer to repair itself as polar stratospheric clouds can't be formed. (I'm not an expert-but it sounds like thats what happening).--Gilisa (talk) 19:40, 28 October 2009 (UTC)
The ozone layer does repair itself, once the sun starts shining on it again. Graeme Bartlett (talk) 19:43, 28 October 2009 (UTC)
What do you mean?--Gilisa (talk) 19:45, 28 October 2009 (UTC)
  • Read Ozone layer#Origin of ozone for how ozone is formed (UV + 02 = 03). A hole forms when depletion happens faster than regeneration. —Akrabbimtalk 20:05, 28 October 2009 (UTC)
  • You got the correlation the wrong way. Ozone is formed whenever oxygen is hit by the proper wavelength of UV light (i.e. when the sun is shining). It is destroyed by various processes, but in particular by those involving chlorine (from CFCs) and polar stratospheric clouds. Polar stratospheric clouds form when it is cold enough. That's why the ozone hole grows in winter (no sunlight, many clouds) and shrinks in summer (fewer clouds, and a lot of sun). Global warming will actually lead to a slight decrease in stratospheric temperature, but the effect should be negligible. And since we have phased out CFCs in the Montreal Protocol, things have slowly started to improve. --Stephan Schulz (talk) 20:13, 28 October 2009 (UTC)
  • Ok, now I see that you wrote ".. it's because the chemical reactions that break down ozone are very much more efficient if polar stratospheric clouds catalyze the reaction". I previously missed the "break down". Thanks for the explanation.--Gilisa (talk) 20:27, 28 October 2009 (UTC)
Some facts:
  1. Ozone depletion happens to some degree over the entire planet - not just at the poles.
  2. Ozone in the upper atmosphere is created by UV light.
  3. There is much less UV light at the poles because they have no sunlight for maybe 3 months of the year - and much less sunlight than elsewhere in the world for the rest of the year.
Because they have less ozone to start with, the general ozone depletion over the entire planet affects the polar regions much more than anywhere else. So even though ozone-depleting gasses like CFC's may be denser over more occupied parts of the world, there is more than enough at the poles to remove all of the ozone and to create a hole. As others have pointed out, factors such as the temperature and the nature of polar clouds tends to exacerbate this problem.
Ozone is a weird problem. At ground level, ozone is a nasty poison and one that we go to great lengths to avoid making. In the upper atmosphere, it blocks harmful radiation and saves us from all sorts of nasty skin and eye conditions. This confuses some people. "Yeaaah Ozone! Boooo Ozone!"
It's worth mentioning that prompt action in phasing out CFC's and other ozone-destroyers has caused a significant improvement in the polar ozone levels - this is an encouraging thing since it suggests that humanity can get it's act together to fix global problems. Now if only we could do the same thing with CO2 and methane - we'd all be riding along on a much better planet.
SteveBaker (talk) 13:12, 29 October 2009 (UTC)

Electron configurations of rhodium and tungsten[edit]

Rhodium's electron configuration is [Kr]5s14d8, rather than the expected [Kr]5s24d7. While several other transition metals have similar anomalies, they have arrangements that either have two shells half-filled (such as chromium) or one shell half-filled and another shell full (such as copper). Rhodium's electron configuration doesn't fit either of these. Also, why is tungsten's electron configuration [Xe]6s24f145d4, when the same reasoning that applies to chromium should apply to it as well? ----J4\/4 <talk> 16:27, 28 October 2009 (UTC)

From your question it looks like you've read Electron configuration article already; please read it if you haven't. Here is a more detailed answer. In atoms of d-block elements, you have three possible ground configurations: (n+1)s2ndk, (n+1)s1ndk+1, and (n+1)s0ndk+2. These three configurations are split into LS terms, which in turn are split into states. The configuration-average binding energies for the three configurations are quite similar; the difference in binding energy between (n+1)s2ndk and (n+1)s1ndk+1 is usually smaller than the differences in binding energies between some of the terms of each configuration. So it may happen that the ground state (the strongest-bound eigenstate) belongs to either one of those configurations, depending primarily on the LS terms allowed for the given value of k by the Pauli principle. Also, binding energies depend substantially on the (fractional) probability to find an outer ("optical") electron close to the nucleus, where coulomb attraction is strongest. This probability is very different for different values of the nucleus charge (different elements), so the d-block elements in periods 4, 5, and 6 may not fill out their outer shells in the same order. For example, Niobium has ground state belonging to configuration 5s14d4, but Tantalum has 6s25d3 and not 6s15d4. Complicating matters further, the three configurations (n+1)s2ndk, (n+1)s1ndk+1, and (n+1)s0ndk+2 have the same parity, so the true eigenstates may be mixtures of states in those configurations; that is, electron wavefunctions are sums of components from different LS terms of different configurations. In such a case it is not even possible to unequivocally determine which configuration is indeed the ground one. --Dr Dima (talk) 19:20, 28 October 2009 (UTC)
(continuing - hit Save instead of Preview. Sorry). So for your first example, Cobalt is 4s23d7 but Rhodium is 5s14d8 and not 5s24d7. That is, very roughly, because the difference in Coulomb interaction with nucleus in Co (Z = 27) 4s and 3d orbital is not the same as difference in Coulomb interaction with nucleus in Rhodium (Z = 45) 5s and 4d orbitals. There are other effects at play, too: difference in electron-electron interaction terms, stronger relativistic effects in Rh than in Co, and so on. The bottom line: terms of 4s23d7 overlap with terms of 4s13d8 in Co, and terms of 5s14d8 overlap with terms of 5s24d7 in Rh; it just so happens that the lowest energy term is not in the same configuration in the two cases, for the reasons explained above. The same applies to Tungsten vs Chromium and Molybdenum, as well. --Dr Dima (talk) 19:36, 28 October 2009 (UTC)

Spectroscopy[edit]

I'm working on the bio of a chemist, Basil Weedon, I want to link to an article detailing what type of spectroscopy he used to elucidate the structures of carotenoids. The source says he used proton magnetic resonance but I can't work out what this actually is to link to, if someone can provide a link I'd to what this is called on here I'd be greatful. Thanks for your help. Smartse (talk) 18:19, 28 October 2009 (UTC)

Magnetic_resonance_imaging#Magnetic_resonance_spectroscopy. --Sean 18:31, 28 October 2009 (UTC)
In particular, NMR spectroscopy. It's the standard chemists' tool for determining molecular structure. I see that proton magnetic resonance has become a bluelink to an article with lots of details about the specific types of this analysis. DMacks (talk) 18:50, 28 October 2009 (UTC)

Planets shrinking[edit]

Is all planets lossing size in diameter or is just Mercury? Is Uranus and Pluto alos losing size in diameter?--209.129.85.4 (talk) 20:08, 28 October 2009 (UTC)

The article says: "3 miles of its 9,000 mile diameter in four billion years" - that's 0.001 of a millimeter per year - a REALLY slow rate of loss. I doubt we have good enough images of the outer planets to measure to that kind of precision. I'm skeptical that we can measure Mercury to that precision either! Whoever says this is true must have inferred it indirectly somehow. SteveBaker (talk) 20:32, 28 October 2009 (UTC)
I believe the surface has wrinkled as it shrank and that is how we know. I think the gas giants are shrinking, at least Jupiter is - from Jupiter: "This additional heat radiation is generated by the Kelvin-Helmholtz mechanism through adiabatic contraction. This process results in the planet shrinking by about 2 cm each year." --Tango (talk) 20:45, 28 October 2009 (UTC)
Yeah, found it - Mercury (planet)#Plains: "One unusual feature of the planet’s surface is the numerous compression folds, or rupes, which crisscross the plains. As the planet’s interior cooled, it may have contracted and its surface began to deform, creating these features." --Tango (talk) 20:47, 28 October 2009 (UTC)
The rate must be even slower than the linearly-interpolated 1 micron per year because as the planet cools, the litospheric thermal boundary layer increases in thickness and insulates the inerior. The ridges are thought to be signs of contraction, but this is an inference from Mariner images; we'll learn a lot more from Messenger soon. Awickert (talk) 02:46, 30 October 2009 (UTC)

IR spectrum of benzophenone[edit]

I will admit first off that this does constitute part of a non-assessed assignment. I have spent ages trying to work this out and I think I may just be having a blonde moment. Basically I have an IR spectrum of benzophenone with percentage transmittance on the y axis and I have managed to identify the origin of most of the peaks. However, there are three peaks around the 2000cm-1 mark. Specifically at; 1977.64, 2028.75 and 2159.88. Thank you for any help.Alaphent (talk) 20:20, 28 October 2009 (UTC)

Infrared spectroscopy correlation table may be helpful to you; though I admit I cannot find any relevent peaks in those ranges that would be expected from benzophenone. My best guess is that the peaks are due to the presence of an impurity of some sort, either left-over starting material or a decomposition product or something like that. --Jayron32 02:02, 29 October 2009 (UTC)
Here's a quick google hit for the spectrum: [16], and I don't see any significant peaks like the ones described. DMacks (talk) 02:09, 29 October 2009 (UTC)

Ok. Thanks, I'd found the quiz from the uni of colorado. It must be an impurity then. The guy who set this would do a thing like that just to be awkard. Alaphent (talk) 19:57, 29 October 2009 (UTC)

Are we Europeans all Jewish?[edit]

It seems that if you go back far enough, everyone is a descendant of everyone (more or less) alive hundreds of years ago. According to TV (huh!) Jewishness passes through your mother and is not extinquished even if you never follow the religious rights. Does that mean that everyone of European descent is technically Jewish? 92.29.91.83 (talk) 20:47, 28 October 2009 (UTC)

No. While if you go back far enough everyone is descended from everyone else (Identical ancestors point), you aren't necessarily related to them through the maternal line. You only have one female ancestor in each generation that is just through the maternal line (your mother, your mother's mother, your mother's mother's mother, etc.). Only if one of them is/was Jewish, or you've converted, will you be Jewish under that rule. There is a very good chance that that won't be the case. --Tango (talk) 20:52, 28 October 2009 (UTC)
That said, I expect there plenty of people that are Jewish under that rule without knowing it, just not everyone. --Tango (talk) 20:54, 28 October 2009 (UTC)
Where did you get from the knowledge that all Europeans have Jewish heritage? I know about recent genetical studies in European Iberian population which reveald about 20-30% of Iberian people are descendant from Jews (on their paternal Y heritage if I remember it right), probably of the Anusim (and additional 10% from Arabs). I also heard that many of Russian people in Moscow descendant from Jews-but still there is a great distance from this to what you just suggested. Also, keep in mind that if X precents of Europeans descendant from Jews no more than X/2 are of Jewish maternal heritage. It was estimated once that without the holocaust and the high rates of assimilation and with the reproduction rates of Jewish families during the 1930's Jewish people would count more than 90 million people by now. Before the mid 19 CE rates of intermarriage of Jews with non Jews population were very very low and assimilated Jews have the reproduction rates of non Jewish European families (i.e., they can't exceed their initial precent in European population this way). So you can make the simple calculation and understand that there is no storng basis for this kind of assumptions.--Gilisa (talk) 21:01, 28 October 2009 (UTC)
That depends very much on when you start considering people Jewish. If (just as an example) the Queen of Sheeba had some children with Solomon, and you accept him as Jewish, they, as part of the nobility, would have plenty of opportunity to spread their genes around. Similarly, I would assume that there would be quite some mixed descendants from the time of the Jewish disapora in Babylonia, some of which may have been in a position to have a lot of children. There is some evidence that Ghengis Khan sneaked his genes into 0.5% of the male world population in just 1000 years, and these processes typically are exponential. I wouldn't be at all surprised if 90% of Europeans have some Jewish heritage within the last 2000 years (and, of course, the same holds for many other people - Romans, Greeks, any of the people culturally and economically integrated in the Roman Empire...). That said: It's a lot easier for males to have a large number of descendants, so for the female line criterion, the chances are a lot slimmer. --Stephan Schulz (talk) 21:23, 28 October 2009 (UTC)
I agree, the vast majority of the European population will have some Jewish ancestors. That is very different to having Jewish matrilineal ancestors. It isn't just enough to have a female Jewish ancestor or for your mother to have a Jewish ancestor, it needs to be women all the way from them to you. --Tango (talk) 21:32, 28 October 2009 (UTC)
...and we haven't even begun to discuss the concept of being "ethnically Jewish," but that is whole 'nother can of worms. Bus stop (talk) 21:41, 28 October 2009 (UTC)
And not one worth opening. Ethnicity isn't a useful way to categorise people; it is extremely imprecise and even if you can define it it says nothing about genetics just culture. --Tango (talk) 21:46, 28 October 2009 (UTC)
I saw an interesting program a while back about how much more related we are than we typically give credit for. The program said that if you went back far enough to a human that had a reasonably large number of offspring, eventually practically everyone in the world is at least somewhat related to them. The example they gave was Cleopatra. She lived 2000 years ago and had a bunch of children. If you assume this is about 100 generations (average generational gap of 20 years) and go back that many generations assuming you have 2 unrelated parents each generation, you get 2^100 or 1.2676506E10^30 people, more people than have ever lived (and maybe ever will live), an absolutely preposterously impossible number for us to have that many ancestors. The conclusion is that we are related to everyone that was alive at that time and didn't have a lineage that died out. Similarly, everyone alive today has two possibilities: in a few thousand years their line will have either died out (no children or particularly unfortunate children) or will be an ancestor to practically everyone on the planet. If looked at from a suitable number of generations, our "family trees" are actually more of a diamond shape. Of course we are related to some historical figures much more closely than others. They occupy a more prominent place in the diamond... Or if we are sticking with the tree idea, the single person occupies many many more of the spots. I suppose this doesn't apply to people completely isolated from the "main body" of humanity until relatively recently. That would include Native Americans and Australian Aboriginals, at a first guess. TastyCakes (talk) 22:02, 28 October 2009 (UTC)
Indeed. The identical ancestors point for the whole human race is estimated to be around 5,000-15,000 years ago. If you exclude groups that are or were until recently isolated, then it is much more recent (although I can't find any figures...). --Tango (talk) 22:22, 28 October 2009 (UTC)
That's a nice article I didn't know about. I think it puts the idea much less confusingly than I did above... TastyCakes (talk) 22:28, 28 October 2009 (UTC)
Stephan Schulz, Ghengis Khan had thousends of women, his heirs had similar numbers of women and you must remember that world population size was much much smaller during his life time, making founder effect to be highly plausible. As for the Jewish people, most of them are descendant from the same ancient paternal ancestor who lived in the Levant about ~ 4000 years ago and this genetic cluster is still well distinguished from other Middle Eastren populations. Also, about 40% of Ashkenazi Jews are descendant from only 4 matrons who lived in the Levant about ~1500 years ago. While you can find this maternal heritage in very small precents among non Jewish populations in the present days Middle East, you virtually can't find it among non Jewish Europeans. So, it's highly unlikely that 90% of Europeans are descendant from Jews. Also, these processes are growing in exponential rise only when the average number of offsprings is higher than the average number of offsprings in the target population. Usually, I think, population genetics models don't use exponential rise.--Gilisa (talk) 22:15, 28 October 2009 (UTC)
I disagree. It is practically certain that everyone in Europe (indeed, everyone in the world with a couple of exceptions) has an ancestor that practiced Judaism, due to the mathematics outlined above. Of course, the contributions to a person's ancestry may be so small as to be genetically trivial. As pointed out earlier, the matrimonial line requirement changes the story considerably. TastyCakes (talk) 22:22, 28 October 2009 (UTC)
If we assume that the reproduction rate of assimilated Jews and non Jews in Europe was similar-and we have no reason to assume otherwise (infact, as assimilated Jews had free occupations and lived in big cities usually their rates of reproduction were lower than these of non Jewish rural populations in Europe) and that European population grow during the last 100 years in exponential rate then the relative genetic contribution of Jews to the European population remains the same. --Gilisa (talk) 22:32, 28 October 2009 (UTC)
But it doesn't matter if the assimilation rate was even almost completely zero. All it would take is one exception far enough back in time for a Jew to become an ancestor to all Europeans, and the chances of that not happening in the thousands of years Judaism has been practiced is zero. I am not saying that Europeans have mostly Jewish DNA, most will have almost negligible amounts for the reasons you are saying. But all will have some. Similarly (and perhaps more mind-bogglingly), it is absolutely certain that you are related to a Chinese person that lived 15,000 years ago. Even though genetic intermingling was very low due to cultural and geographical separation, it wasn't zero and hence Chinese DNA undoubtedly mixed with all other Eurasians' (and Africans') DNA over the past few thousand years. TastyCakes (talk) 22:38, 28 October 2009 (UTC)
To clarify the basic error in the question, it's true that if you trace back all of a person's ancestors, the number doubles every generation you go back, except for overlap due to multiple paths leading to the same ancestor. That means if you go back a hundred generations, you get a major fraction of the world as ancestors. But if you only count mothers, you only get one for each generation, so for a hundred generations back, you only get a hundred mothers. Looie496 (talk) 22:46, 28 October 2009 (UTC)

By the same logic are we all Muslem? And are all Muslems Jewish and all Jewish people Muslem? 92.24.25.252 (talk) 01:38, 31 October 2009 (UTC)

Maillard reaction creates acrylamide?[edit]

I thought that acrylamide was produced as a result of the Maillard reaction, but the Maillard reaction article does not mention it. Does it or dosnt it create acrylamide please? 92.29.91.83 (talk) 20:53, 28 October 2009 (UTC)

The Maillard reaction article does not mention it, see the page on acrylamide - this does clearly state that that it may be a by-product - not the main product - of the Maillard reaction. It may well be that the Maillard reaction product decomposes to acrylamide with more heat, but that does not make it a product of the Maillard reaction, but a product of a subsequent pyrolysis reaction.  Ronhjones  (Talk) 22:45, 28 October 2009 (UTC)

What is the intermediate product please? 78.144.248.102 (talk) 10:32, 30 October 2009 (UTC)

Icons of Evolution[edit]

I'm about to rip my hair out.

I read the article on Icons of Evolution. Please glance over it. Note that in the first sentence the book is condemned as pseudoscientific. Later a sweeping claim is made that all biologists endorse evolution. Hopefully, even if you are an evolutionist yourself, you can see that that isn't proper for an encyclopedic article. I can provide names of at least three prominent non-evolutionist biologists -- Michael Behe, Dean Kenyon, and Phillip E. Johnson. Don't be silly like Auntie E. and say that just because I provide only three names means that there are only three.

This is my problem. While reading the article, I noticed that the article lacked one huge thing: information on what was said in the book! This article claims to be ABOUT Icons of Evolution, but has nothing on what's discussed in the book. Nearly everything in the article is criticism -- cited criticism, at least, but still it's a very biased article.

You might say, "Well, go fix it yourself." I tried. I don't like to be abusive, but Auntie E. has been downright incorrigible. I eliminated the word "pseudoscientific." I took out the sweeping claims discussed above. Under some chapter headings, I added a little on what was discussed in the book. Did Auntie E. even read what I wrote before deleting it under the pretense that it was not neutral? I honestly don't know.

Please talk with Auntie E. This shouldn't be a question of Intelligent Design versus Evolution. This article should, if you apply common sense, be a pro-ID article, because for heaven's sakes the book that is supposed to be discussed in it is pro-ID! Anyone who tries to clean up the bias finds Auntie E. flaunting neutrality rules in their faces. I wanted to uphold the integrity of Wikipedia. Instead I'm being browbeaten by a biased "editor" who seems, by her conduct, to be interested in nothing but safeguarding her evolutionist beliefs. --Thalia14 (talk) 21:12, 28 October 2009 (UTC)

I'm not sure that this is the right forum, why won't you take it to wikipedia's mediation? --Gilisa (talk) 21:17, 28 October 2009 (UTC)
(Johnson is not anything like a biologist, he's a law professor, argues about the philosophy of materialism, etc.) Anyway, I do agree that the article suffers from quite a bit of POV—it is nothing but rebuttals, aside from one list of Wells' "icons", which really is quite insufficient for a book, even one full of contentious and challenged statements. Anyway... in my personal experience, the best you can do is be calm, try to appeal to more high-minded folks. You are unlikely to have any success, though—Wikipedia's NPOV on such topics usually is the SPOV, even though it isn't supposed to be. Editing contentious Wikipedia articles is basically a game of seeing who is most persistent and who knows how to use the system to their aims better. It is a frustrating and un-fun experience, and the stakes are, in my opinion, not high enough to warrant it. It is one of the reasons I barely edit articles anymore. --Mr.98 (talk) 21:28, 28 October 2009 (UTC)
I suggest you compare it with the Featured Articles, Uncle Tom's Cabin and Jack the Ripper: The Final Solution. Notice how both of these tell the reader very early on whether the work is widely believed to be true or not. They both devote a good deal of space to discussing how the works were received, and how they are currently received, particularly by people who are experts on the topics the books are about. They also include brief information on the key contents of the works: you can find the same in the Icons of Evolution article under Wells' Icons. It's quite a large section.
You need to read WP:NPOV (again, if you already have) and understand that the article should not be pro-ID at all. If your real problem is that you think evolution can't possibly be true, and therefore anything which does not support Wells' criticisms must be dishonest, you need to step back and decide whether you can honestly edit any articles on this topic without getting emotionally involved and violating NPOV. 80.41.80.71 (talk) 21:41, 28 October 2009 (UTC)
I think that she made few good points. Exclusion of information from an article, when there is room for it, is second worse only to inclusion of wrong arguments in it. It doesn't matter what you believe in, objectiveness must be kept and I think that maybe in this case it was missing to certain extent. More, I understand that you didn't discuss it on the talk page? Also, I suggest you to take the advises of Mr.98. Also, remeber that you may use Wikipedia different boards if you feel that edits on the article are being done without agreement. I suggest you to seek mediation by third party.--Gilisa (talk) 21:49, 28 October 2009 (UTC)
Yes, a better place is the talk page, or WP:NPOVN, or a RfC. But I disagree with much of the criticism - starting with the small, I could not find the "claim that all biologists endorse evolution". --Stephan Schulz (talk) 21:56, 28 October 2009 (UTC)
You're both right the first thing to do of course would be to actually try discussing this on the article talk page. I see User:Aunt Entropy has indicated a willingness to do so and it looks like at least one other editor is as well. I suspect if the user actually attempts to discuss it, other people would be willing as well. However attacking other editors is not going to help the discussion so I suggest Thalia refrain from doing either of these if they wish to discuss it further in the talk page Nil Einne (talk) 22:36, 28 October 2009 (UTC)
This is absolutely not the right place to bring up these complaints. APL (talk) 22:24, 28 October 2009 (UTC)
Well well a disagreement between two Scientists called Steve on ending this discussion. I am all for ending it myself. It really is not about a scientific subject. --BozMo talk 13:04, 29 October 2009 (UTC)


May I suggest the Help Desk for editing help? 66.65.140.116 (talk) 00:12, 30 October 2009 (UTC)

Saturn V versus Ares rocket: Reinventing the wheel.[edit]

NASA and the TV news made a big deal of the launch of the new Ares rocket, going on about how amazingly big and powerful it is. Checking the stats, the Saturn V was 110.3 meters tall versus 94 for the Ares 1-x, had a diameter 10.1 meters of compared to 5.5 meters, and could deliver 118,800 kg to low earth orbit compared to just 25,000 kg. The Ares thrust is not stated in the Wikipedia article, but a website says it is "3 million pounds" compared to 7,648,000 for the Saturn. My question: Is the Ares less than 21% of the cost per unit compared to the Saturn V (inflation adjusted)? The remaining development costs are also to be considered for the new rocket. The Saturn also has a demonstrated safety record. Why didn't NASA dust off the blueprints for the old Saturn V and have Boeing build a few? (If the U.S, government threw away the detailed plans, they could probably get a copy of what spies likely supplied to the USSR.) Edison (talk) 22:49, 28 October 2009 (UTC)

They shouldn't be making a big deal about how big and powerful it is. That isn't the important thing about it. You don't just want the most powerful rocket you can get, you want a rocket that is right for the job you intend to use it for. Ares I is just supposed to carry the people up to LEO, Ares V will do the heavy lifting. (Assuming they actually get that far - that is rather in doubt at the moment.) --Tango (talk) 22:55, 28 October 2009 (UTC)
Hopefully this doesn't come across as soapboxing as it is largely OR. I read on one of the article talk pages (found it here Talk:Orion (spacecraft)#Falcon 9 Heavy to replace Ares I?) someone saying most people recommended NASA should have designed something like the Soyuz (after all the Chinese did and it appears to work well for both of them, the Russians and Chinese are fairly secretive about their safety record and costs but I would expect the US government has a rather good idea of both) but NASA ignored them because they didn't want to use something they didn't design. I don't know if this is true, but it does seem likely NASA would find it politically problematic to copy another country particularly the Russians. In the same vein, I would wonder whether just reusing a perhaps slightly modified old design would have similar problems of being not politically acceptable. After all when you have people like this [17]... (Okay the last bit was only of minimal relevance but I've been dying to use that since I came across it a few weeks ago) Nil Einne (talk) 23:36, 28 October 2009 (UTC)

Even if NASA wanted to build more Saturn V's, it wouldn't be just a matter of "dusting off the plans". Components that you could buy off the shelf in the 1960s are not necessarily available at all today, and that includes not only the parts used directly in the rocket, but also the parts used to make the devices used to make the rocket. For that matter, even the Vehicle Assembly Building is no longer available as long as it's needed for use with Space Shuttles. --Anonymous, 23:55 UTC, October 28, 2009.

To some extant they have: Apollo Heat Shield Uncrated After 35 Years, Helps New Crew Vehicle Design and Museums guide NASA to future missions Engineers study Apollo exhibits to learn how to return to moon:

"Snoddy, a manager at NASA’s Marshall Space Flight Center, has been removing valves and other parts from Apollo exhibits as he oversees construction of the upper-stage engine on the new moon rocket, dubbed Ares 1." Rmhermen (talk) 01:19, 29 October 2009 (UTC)

As Anon said, one of the main reasons they did not re-use the Saturn V is because the infrastructure for building it is not in place. NASA was not the only organization involved in the Apollo missions--they contracted out a lot of the R&D and construction to corporations around the country. All of the facilities, personnel, and resources that were used to build the Saturn V are now being used for other purposes that may not even be related to the space program (for the record, the Vehicle Assembly Building actually was used for the assembly of Ares I-X, it's the only building in the world that has that capability). The Constellation program actually does use a lot of old infrastructure, as the OP suggested--a lot of the hardware is based on the Shuttle program! Ares I is adapted from the Shuttles' Solid Rocket Boosters, and the liquid rocket of Ares V is adapted from the corresponding system on the Shuttle. Mildly MadTC 18:16, 29 October 2009 (UTC)

There was no existing infrastructure for creating pumps, combustion chambers, fuel storage tanks, etc for Saturn. It was all created in a very few years. This is a red herring. It could be re-created with the benefit of 20-20 hindsight. As I asked initially,, is the Ares proportionately cheaper to its lower earth orbit capability? Recovery of the solid rocket casings, compared to throwaway of the Saturn booster is not a convincing factor in favor of Ares, absent cost information. The ability of the Shuttle to land and take off again was supposed to make spaceflight super cheap, but it did not turn out that way. Edison (talk) 04:56, 30 October 2009 (UTC)

tiredness[edit]

why is it that at bedtime I feel sleepy but if i stay up half an hour past my usual bedtime I'm then not sleepy? —Preceding unsigned comment added by 86.166.235.46 (talk) 23:16, 28 October 2009 (UTC)

Its probably a consequence of your circadian rhythm. If you maintain a regular bedtime schedule, its likely your body has become entrained to certain zeitgebers that define your bedtime. You then upset that schedule, your rhythm goes out of whack, and the molecules in you body that makes you feel sleepy are no longer there. Rockpocket 00:04, 29 October 2009 (UTC)
The MOLECULES in your body??? Which particular ones? What sort of answer is that? This is a non answer of the highest magnitude. —Preceding unsigned comment added by 79.75.3.92 (talk) 02:20, 29 October 2009 (UTC)
As Looie496 correctly points out below, the molecular mechanisms involved in sleep are not yet fully understood. While melatonin is clearly a big player in chronobiology, its unlikely that it is primarily responsible for the observation the OP is asking about. Nevertheless, anyone who has experienced jet-lag can attest that circadian rhythm and the feeling of tiredness/alertness have a close relationship. Rather than speculate on the precise molecules involved, I preferred to offer the OP an general overview of circadian rhythm (with links) and how it would effect sleep. If s/he or anyone else would like a more detailed answer, with links to primary sources, then I would be happy to offer them. Other than that, I would note that my answer was orders of magnitude more constructive than the childishly petulant outbursts of a certain banned former editor. Rockpocket 18:49, 29 October 2009 (UTC)
The primary molecules involved in the management of circadian rhythms usually include Melatonin. The idea is that melatonin levels wax and wane, and your body's response to it waxes and wanes as well. If either your melatonin levels drop, or if you become tolerant to existing melatonin levels, you may "miss" the opportunity to fall asleep. --Jayron32 03:41, 29 October 2009 (UTC)
"This is a non answer of the highest magnitude." Actually, it was a rather informed answer that Rockpocket generously took the time to provide. Nobody tells us that we have to answer questions here. Falconusp t c 03:51, 29 October 2009 (UTC)
Well, it was probably a guess, actually -- the relationship between circadian rhythms and sleep patterns is not that precisely understood. Another possible answer is that the questioner tends to engage in behaviors that are more stimulating on the occasions when he misses his usual bedtime. The melatonin-based explanation is pretty unlikely -- there are much more direct interactions between the circadian system and the brain circuits that control sleep. Looie496 (talk) 05:00, 29 October 2009 (UTC)
Becoming aware that one is missing one's normal bedtime and possibly anticipating that getting up the next morning will be difficult is a stress situation where the nervous system may generate Epinephrine (adrenalin) hormone which is a "fight or flee" stimulant.Cuddlyable3 (talk) 10:38, 29 October 2009 (UTC)
You may be addicted to caffeine. If you abstain from caffiene then you will feel not so good for a few days, but then sleep very well and wake up feeling refreshed and alert all the time (as if you had just had a large coffee). 78.144.206.114 (talk) 11:27, 31 October 2009 (UTC)
Whoah there 78.144.206.114. You gave medical diagnosis and advice which we DO NOT GIVE. Cuddlyable3 (talk) 14:55, 31 October 2009 (UTC)


October 29[edit]

Effect of smokeless tobacco on dental health[edit]

What effect, if any, does smokeless tobacco (in general) have on dental health? Please answer from a dental practitioner's perspective, if possible. Thanks, The Ace of Spades(talk) 01:52, 29 October 2009 (UTC)

Wikipedia does have articles on smokeless tobacco, however looking though them, especially the one on Dipping tobacco, it appears the article has been entirely whitewashed by tobacco apologists. There are no actual links to any hard, independent studies into the health effect of dipping; the entire health section merely spends its time refuting studies which it gives no space towards reporting the results on. Sadly, Wikipedia may not be the best place to get accurate information on this. --Jayron32 01:58, 29 October 2009 (UTC)
Nevermind, managed to find something elsewhere, thanks for the tip on that. Though this may not be the place to discuss it, should the dipping tobacco article be tagged with anything because of the said whitewashing? The Ace of Spades(talk) 02:13, 29 October 2009 (UTC)
Although the overall lifetime risk of malignant transformation in chronic users of smokeless tobacco is <1%, smokeless tobacco is carcinogenic and in addition to being associated with squamous cell carcinoma, verrucous carcinoma is uniquely associated with the use of smokeless tobacco. Additionally, smokeless tobacco may lead to gingival recession and is a modifying factor in periodontal disease, halitosis, staining and a possible increase in dental caries. Harmful constituents are readily absorbed into the oral mucosa with which they are in direct contact. Oral submucous fibrosis, of similar pathogenesis, is related to the chronic use of betel quid/areca nut, used by people in/of India, Pakistan and surrounding areas of Southeast Asia. Of submucous fibrosis cases, approximately 10-15% show epithelial dysplasia and roughly 8% exhibit squamous cell carcinoma. (All info from oral pathology course lecture notes, NJDS) DRosenbach (Talk | Contribs) 02:37, 29 October 2009 (UTC)
Thanks so much, that's exactly what I needed. Better than what I found online. The Ace of Spades(talk) 02:44, 29 October 2009 (UTC)

Human Hearing Range in Nature[edit]

Are all the frequencies in the human hearing range (12Hz-20KHz) present in humans' habitable biosphere, apart from what's produced by recent technology? If not, why/how did humans evolve to be able to hear them? --CodellTalk 04:55, 29 October 2009 (UTC)

While lacking the time to find and cite sources, I think it is safe to say yes, all frequencies between 12Hz-20KHz occur in the natural world humans live, and lived in. Pfly (talk) 09:32, 29 October 2009 (UTC)
Only a small range of frequencies about 300Hz - 3kHz is really needed to communicate using speech (e.g. used in telephones) and human hearing has evolved greatest sensitivity in this range, arguably because of the survival value of a parent hearing the cry of their baby.Cuddlyable3 (talk) 10:29, 29 October 2009 (UTC)
Greatest sensitivity maybe in that range, but a telephone quality sound is hardly high. Perhaps the ability to hear higher frequencies results from these other frequencies being present as harmonics? I have no evidence to back this up its just a thought. —Preceding unsigned comment added by Alaphent (talkcontribs) 14:25, 29 October 2009 (UTC)
Hearing higher frequencies has survival value because they aid localization of sound sources. Many sounds have harmonics beyond the human hearing range, the upper limit of which drops way below 20kHz as one ages.Cuddlyable3 (talk) 19:57, 29 October 2009 (UTC)
The sound of the ocean coast is an extremely good source of white noise which contains that whole range of frequencies. I'm thinking wind in a forest would too, maybe wouldn't go as low as ocean but it would probably still pass. Vespine (talk) 21:43, 29 October 2009 (UTC)
So maybe being able to hear the white noise of the coast and wind through trees would be beneficial for humans as both were a source of food. Might cultures who have lived for a long time in desert regions therefore have a lower upper hearing range? --CodellTalk 00:09, 30 October 2009 (UTC)
Well first of all, humans didn't actually evolve hearing so I think that's where your first misconception lies. Hearing has been around a long long time since before humans so it's quite probable we just inherited our range from our ancestors. Also, don't fall into the trap of thinking that every single observable trait in every species must have been the result of some selective pressure or evolution. Amongst others there are traits which are vestigial which are seemingly useless traits left over from past evolution, like goose bumps, if there is no real pressure to get rid of the trait there is no reason for it to disappear if it is not very costly to the organism, and also there are associated traits which can also seem useless but are somehow genetically linked to other seemingly unrelated traits which do have a good use. Vespine (talk) 04:28, 30 October 2009 (UTC)
Good point. I'll also note that hearing higher frequencies could have survival value because it helps us discriminate between different sounds. You can tell a violin from a trumpet when they play the same note because the overtones are different -- the same might apply to sounds in nature that it's desirable to tell apart. --Anonymous, 19:09 UTC, October 31, 2009.

Counter circuit Photo-Electric Beams[edit]

I want to design a counter using laser beams. The purpose is such that if someone enters the room they are added to the list. If they move out the it they are removed from the count. If the count is 0 then it switches off the lights, if it is more than 0 it switches on lights.--yousaf465' 08:11, 29 October 2009 (UTC)

You might construct your counter by soldering some logic integrated circuits on a small circuit board. The counter itself could be a 4516[18] which being 4-bit limits the count to 15 7 persons. If you need to store larger numbers of persons use more counter stages. Detecting whether a person comes in or out needs two photocells and lasers. With a 4013 dual flip-flop you can store the first photocell pulse then the second pulse clocks the counter and the order of the pulses determines the count direction up or down. The empty=0 state of the counter controls the lights. This logic circuit runs from say 5V so it needs interfaces to the small signals from the photocells, possibly made with op-amps[19] like 741Operational amplifier, and to the lamps. If the lamps are mains powered then you need an optoisolator[20]+triac[21] or a relay to control them safely from your 5V logic signal. When the counter is full=15 7 or empty=0 it must be disabled from counting further in the relevant direction. The references give more information and there are many alternative components.Cuddlyable3 (talk) 10:23, 29 October 2009 (UTC)
Thanks for the answer. I will look into it.--yousaf465' 12:46, 29 October 2009 (UTC)
The usual way to achieve this is to install a cheap passive infrared sensor in the room rigged up to switch off the lights if it detects no movement for, say, seven minutes. You find such systems in public buildings etc, so they must be available off the peg.--Shantavira|feed me 16:00, 29 October 2009 (UTC)
What if I want to sit alone in the OP's room and read a book for 8 minutes? Cuddlyable3 (talk) 19:52, 29 October 2009 (UTC)
Then every 7 minutes you wave your hands around like a maniac when the lights go off. (It's how it is done in universities, which often have such features.) --Mr.98 (talk) 00:40, 30 October 2009 (UTC)
This will help with exercises. --yousaf465' 02:52, 30 October 2009 (UTC)
  • How should it be stopped from counting further if it's empty ?--yousaf465' 02:57, 30 October 2009 (UTC)

Please see my circuit below. I limited the counter to 7 persons so that Q"8" limits the up count at 7, and CARRY limits the down count at 0. Cuddlyable3 (talk) 21:09, 30 October 2009 (UTC)

I'm sorry to say that what you propose won't work. With just one beam you can't tell whether the person is arriving or leaving - and you can't tell when someone steps into the beam - changes their mind and steps back again. You need at least two beams - then the order of breaking and unbreaking of the beams will tell you which direction the person crossed the beam - and whether they changed their mind again. But this is still not perfect - when one person enters at the exact same time that another person leaves - you can't know what happened.
At the heart of the problem here - if your counter EVER gets confused - even once - it'll either turn the lights off when one person is inside the room (and be extremely difficult to pursuade to turn them back on again) - or it'll turn the lights on with nobody in the room and refuse to ever turn the lights off again.
The difficulty with such systems is that you are trying to measure an absolute quantity (the number of people in the room) using a relative measurement (the number of people entering and leaving) - and any system like that that has any possibility of error whatever will gradually accumulate error until it's useless. In your case, it'll happen rather quickly...the very first time two people pass in the doorway for example. Even if you could sort out that case, you also have the problem if the power fails and your system has to restart - because you have no idea how many people are in the room to start with.
So at a minimum, you'll need a switch to say "Stoopid computer - you turned the lights off and I'm still here" or "Hey - turn the lights off, there's nobody in here!" - and perhaps some logic to (for example) turn the lights off after they've been on for a very long time. At least that provides some means for the system to recover from occasional errors.
The simplest way to implement the complex logic required would be to use a little 'Arduino' computer - you can buy one all ready to run on a circuit board for $25 - and if you can make your own circuit-board, you can buy just the chip for less than $5. You can write simple programs for it on your PC in C/C++ and download them into the computer using a USB port. With software involved, the complicated rules about breaking and unbreaking beams would be easy to set up. You could do timing and add software to try to decide when things have gone astray. You could even add a photosensor to turn the lights off during the day. You could do it with hard logic components - but you'll never do that for less than the cost of an Arduino chip.
A much, MUCH better solution would be something like a burglar alarm - something that detects motion or body heat or sound or floor pressure or something. Those are absolute measurement devices - not relative like a "number of people who went through the door" detector. Of course a cat can fool a motion detector, leaving the TV turned on in the room can generate enough heat to fool a heat sensor - ditto with sound detectors - and moving the furniture around can fool a pressure sensor.
This is actually a really tough problem to solve - which is probably why such systems aren't fitted to every home on the planet.
At our offices at work, we have a RFID tag system that does that. You have a little plastic tag that you put in your wallet - it gives you access to the front door and to the elevator - and when you're within range of any of the detectors scattered around the offices, the lights in the area come on. When you aren't, they go off. If nobody is near a detector, the airconditioner turns off too. It works fine - just so long as you don't lose your RFID tag. My car (a MINI Cooper'S) has a similar system. You just need the circular "remote" (it's not really a "key") in your pocket and it unlocks and locks - and allows the engine to be started with a simple push-button - and it'll turn off the radio and the lights and lock the doors when you get more than a few feet from the car.
Having people carry something that can be detected uniquely at a distance is the answer.
SteveBaker (talk) 12:53, 30 October 2009 (UTC)
Yes I was also looking for a 2 beams system. I will upload the diagram how I visualized it. Also for washroom we can even eliminate the counter, after all you can't have more than one person in a washroom. :)

Well for power I will be using rechargeable batteries.

I'm also interested in RFID but for my cat. How much that system cost. --yousaf465' 04:40, 30 October 2009 (UTC)

Any Fritzing diagram could be helpful.--yousaf465' 05:06, 30 October 2009 (UTC)
The problem isn't going to be fixed with two beams - or three or four for that matter. There are just to many ikky cases that can cause your system to miscount - and it only has to miscount by ONE and it'll be wrong forever more (unless you reset the machine somehow). Because people don't neatly cross beams like that (they may break it first with one arm, then their body and then the other arm, for example - or someone may wave their hand through the beam to see what it does - or they might be wearing a thick, fuzzy sweater that gradually breaks the beam causing a lot of rapid on/off pulses that could easily be miscounted. Any one of these behaviors (and worse-still, any of the two dozen others that we haven't thought of) will cause at least a single miscount - and that's enough to leave you sitting alone in the dark - or having your light left on 24/7. It's not just a tiny problem that you can just blindly ignore - it's the total downfall of the entire proposal. Getting rid of the counter for a single-occupancy area doesn't help that - all it is act as a one-bit counter...it can still get confused and once it's in the wrong state, there is nothing to correct that error. In short: IT WON'T WORK! SteveBaker (talk) 12:52, 30 October 2009 (UTC)
What about any system which works.--yousaf465' 13:01, 30 October 2009 (UTC)

A computer with a webcam and face recognition software (or perhaps recognition of more general features such as clothing or height) could be made to recognise individuals, ambiguous entry and exits and count them correctly. Trevor Loughlin13:39, 30 October 2009 (UTC)

That'd be a rather clear-cut violation of the KISS principle. Red Act (talk) 15:18, 30 October 2009 (UTC)

What I think would work great would be a motion sensor in combination with a one-beam doorway sensor. The motion sensor should be positioned such that it doesn't quite cover the doorway. The system keeps the lights on, unless the doorway sensor was triggered more recently than the motion sensor, and it's been at least 7 minutes since the doorway sensor was triggered.

A system like that would be better than just a motion sensor, in that it isn't necessary to at least wave your arms every 7 minutes in order to keep the lights on. And it's better than just a doorway sensor, in that there's no count maintained that could easy get off by one.

With a two-sensor system like that, if the system does wind up in an incorrect state, it's easy to fix without needing any additional user interface. If the lights are off incorrectly, because you've been sitting motionless in the room for seven minutes after someone else left the room, you just wave your arms to turn the lights on and put the system back into a correct state. Or if the lights are on incorrectly, because someone thought it'd be funny to leave the room by crawling under or stepping over the beam, you just walk into and out of the room to put the system back into a correct state, and the lights will go out seven minutes later. Red Act (talk) 14:47, 30 October 2009 (UTC)

I agree that's a better approach - you need some kind of absolute way to determine if there is someone in the room. Incrementing and decrementing a counter is not the answer. The problem is that humans are tough to recognise electronically. A curtain blowing in the wind from a partially opened window - or a rotating ceiling fan will trick most motion detectors. Pets will also trigger them. I agree that using a beam-breaking system in conjunction with the motion detector will work much better than either of them alone - but I strongly suspect you'd still find the light turned on when it shouldn't be a bunch of times. The only approach that I'm aware of that's used 'for real' is the RFID tag system. That (in effect) modifies the concept of "what is a human?" to be something that computers can easily understand. Of course it's still gonna fail if everyone keeps their RFID tag in their wallet - and then leaves the wallet on the table when they leave the room. SteveBaker (talk) 16:33, 30 October 2009 (UTC)

Approach #1: Treat the OP with WP:AGF. He wants laser BEAMS. Approach #2: Tell the OP they don't really want what they say they want and instead try to sell them these. I'm doing Approach #1, see image. Cuddlyable3 (talk) 21:02, 30 October 2009 (UTC)

Room Occupant Counter Cuddlyable3 (talk) 21:02, 30 October 2009 (UTC)
Well I would like anything which works, not only laser beams. Red Act's approach could be more useful if a simple diagram could be shown.--yousaf465' 04:06, 31 October 2009 (UTC)
In that case install a false floor resting on rubber blocks. When someone is in the room their weight depresses the floor that operates a microswitch that turns on the light. Cuddlyable3 (talk) 14:51, 31 October 2009 (UTC)
Well I don't want my floor to be damaged. I have to do it practically.--yousaf465' 13:13, 1 November 2009 (UTC)

Raisin aversion[edit]

There are cereal mixtures with "no raisins added". Food aversion (exception allergic reactions, toxicity) are socially transmitted (e.g. food taboos). Is there a biological reasion why some people dislike raisins? --Grey Geezer 11:39, 29 October 2009 (UTC) —Preceding unsigned comment added by Grey Geezer (talkcontribs)

Wait. Do you really think "disliking raisins" is the same as a "food taboo"? I think you are mixing raisins with apples here. If you instead want to ask "why do different people have different likes and dislikes (including food)?", it all ends up imho in a "nature vs. nurture" debate. Some genetic differences might be present (e.g., in olfactory receptor equipment of the individual in the case of food), but the individual experience also has an impact of what we learn as pleasant or unpleasant, in an often unpredictable way, for example, because subconscious perception can influence our "wiring" in the brain without us ever knowing. --TheMaster17 (talk) 12:19, 29 October 2009 (UTC)
Food aversion is not just socially transmitted. Variation in food preference is part of behavioural diversity which is a portfolio evolutionary strategy, especially when you are not talking about main staple diet. It is not obvious in evolutionary terms that raisins pose no risk to people, after all they pose a risk to dogs see Grape and raisin toxicity in dogs, are stale food and look like animal droppings. So some of us being programmed not to gorge them on sight is perhaps safer for humanity than if we were all inclined to love them to bits. --BozMo talk 12:22, 29 October 2009 (UTC)
It was not about "disliking" it was about "aversion" ("I can't eat that!"). Found another article where there was an association with "an animals teat" (but how many children have actually seen an animals teat?). So I settle for "Looks actually like a ... naah! I can't eat that!". Case closed. Grey Geezer 13:10, 29 October 2009 (UTC) —Preceding unsigned comment added by Grey Geezer (talkcontribs)
One of the most extensively used experimental procedure in memory study (at the molecular level) in rats is CTA. It appears that if rat experienced physical bad feeling after tasting new food it will never taste it again, as long as the feeling starts in no more than 8 hours delay from eating it. This is also the answer for why seek rats avoid eating, i.e., it will make wrong connection between good food and its seekness. And this is also the reason for which rat poison is slow working --Gilisa (talk) 13:13, 29 October 2009 (UTC)
We humans are so smart. Bus stop (talk) 13:20, 29 October 2009 (UTC)
Nope, it take us more times to develop CTA.--Gilisa (talk) 13:27, 29 October 2009 (UTC)
What does "CTA" stand for? Bus stop (talk) 13:34, 29 October 2009 (UTC)
Contidioned Taste Aversion.--Gilisa (talk) 13:37, 29 October 2009 (UTC)
Wow, we already have an article on Conditioned taste aversion. Bus stop (talk) 13:40, 29 October 2009 (UTC)
With a lot of the up to date information missing. --Gilisa (talk) 14:10, 29 October 2009 (UTC)
It could be due to adaptive behavior like CTA. As for a specific raisin mechanism, I highly doubt it's known. Food aversion has been studied quite a bit, but a true mechanism still isn't clear generally. It's been studied in many animals, and in humans with a wide range of conditions like autism (where it's very common), anxiety (PMID:15576070), and even after certain surgeries (PMID:16925376). This all indicates what you might expect, that there is a physiologic reason. - Draeco (talk) 13:49, 29 October 2009 (UTC)
Actually, we do know that blocking specific dopamin receptors would prevent the development of CTA. However, what we yet don't exactly understand is how the retrieval mechanism of CTA works (but even here we prograss in huge steps - Yadin Dudai's studies from 2007 and on are focused on experimental extermination of already acquired CTA) .--Gilisa (talk) 14:20, 29 October 2009 (UTC)
Hello Gilisa, thanks VERY MUCH for this piece of evidence. This could explain, why a tasty nutrient is refused (because there was an unpleasant experience along with it.) Thanks again! Grey Geezer 14:26, 29 October 2009 (UTC) —Preceding unsigned comment added by Grey Geezer (talkcontribs)
For nothing :)--Gilisa (talk) 14:38, 29 October 2009 (UTC)
Sure. If you don't like raisins than you can be sure that they contain compounds that do not agree with your biochemical system. Vranak (talk) 14:52, 29 October 2009 (UTC)
It's enough that you only change, or even camouflage, the compounds that are responsible for the flavor of raisins for one to eat them without feeling any dislike. Once the food is within your body it's much harder for it to decide what it's, the only thing your body can tell at this stage is whether this food have nutritious value and how it effect on visceral feeling.--Gilisa (talk) 15:04, 29 October 2009 (UTC)
There's a very simple reason why some people dislike raisins: they are quite bitter. Their sweetness makes this hard to recognize, but people who especially dislike bitterness will still be sensitive to it. Looie496 (talk) 16:28, 29 October 2009 (UTC)
And here you suggest that humans are genetically programed to prefer certain tastes with individual differences. That's another good option for why our friend dislike raisins.--Gilisa (talk) 17:06, 29 October 2009 (UTC)

We have a voluminous article Taste that links to Acquired taste but not to Conditioned taste aversion. I like the smart human Bus stop did not recognize the abbreviation CTA. That seems an unhelpful addition to this alphabet soup that could be avoided by merging the article on aversion into a section here titled "Acquired distaste". Cuddlyable3 (talk) 19:34, 29 October 2009 (UTC)

Some people get quite ill if they eat fructose beyond a certain proportion to glucose consumption. Eat a lot of fruit and get stomach pain, diarrhea, fatigue and depression. Large amounts of raisins (or apples, fruit juice, honey, or corn syrup, etc. ) would be bad for such persons. See Fructose malabsorption. It is not a matter of taste preference so much as a metabolic incapacity. Edison (talk) 04:49, 30 October 2009 (UTC)

Earth spinning[edit]

There's no "great power" actively holding the Earth spinning around itself and orbiting the Sun at the exact same speed for eternity, right? It just happens to be at the current speed because of various physical properties such as kinetic energy. Is this right? If so, then is the Earth's speed actually slightly changing? If yes, by how much? JIP | Talk 19:32, 29 October 2009 (UTC)

The great power is gravity and if the sun and earth were solid masses orbiting in a vacuum then Kepler's laws of planetary motion and the Earth's inertia would keep the system running "for eternity" with energy conserved. In reality the sun and earth are not solid masses. Both tides on Earth and the Sun itself expend heat energy and the Earth/Sun system is not isolated from external disturbances such as asteroids and comets. So nothing is forever. Cuddlyable3 (talk) 19:47, 29 October 2009 (UTC)
You'll want to read Earth's rotation. The speed of the rotation has definitely changed over the years due to Tidal forces from the Moon. ~ Amory (utc) 19:50, 29 October 2009 (UTC)
For numbers pertaining to the Earth's slowing rotation, see Tidal acceleration#Quantitative description of the Earth-Moon case. The Earth's orbit around the sun is also slowing down; see Year#Variation in the length of the year and the day. Red Act (talk) 20:08, 29 October 2009 (UTC)
It seems like you already got your answers, but just to put it in simpler words: As for earth spinning itself, the law of Angular momentum preservation keep body spinning around itself (around its center of gravity) as long as angular force is not operated on it and its shape remain constant.
As mentioned here already, tiadl forces against the direction of earth spinning are operating on earth so the spinning is being decelerate by 2.3 miliseconds once in 100 years.
Amory already mentioned that as for Earth orbiting around the sun - body would keep moving in direct line and in constant speed as long as other force is not operating on it and Earth orbiting the sun according to Kepler's laws of planetary motion.--Gilisa (talk) 10:52, 30 October 2009 (UTC)
Taking into account general relativity, even an isolated two-body system will lose energy as gravitational waves - but as you can read in the article, that's indeed very little for the Sun-Earth system. The current changes in the orbital parameters (i. e. their first derivatives with respect to time) you can find here. JPL's solar system dynamics site provides other interesting data as well. Icek (talk) 20:20, 30 October 2009 (UTC)

How many buildings 2 story and above are there in the U.S.A.[edit]

I would like to know how many buildings taller than 2 stories are there in the U.S.A. Completed and/or under construction, if at all possible. But mostly how many buildings in all 2 stories and above. Thank You Very Much. --76.122.225.119 (talk) 21:03, 29 October 2009 (UTC)

I very much doubt you will get a good answer to this. Two-story is a very common configuration for houses and there are a lot of those. Also planning and approval is a local government duty (and some few places isn't always required), not federal or even state. Rmhermen (talk) 00:56, 30 October 2009 (UTC)
If good estimates of this do exist, they are probably hidden somewhere in the US Census Bureau - Manufacturing, Mining, and Construction Statistics site. (E.g., here is a table of new residential construction from the 1970s-present that breaks things down into stories—not quite the same thing as asked but it does give an estimate of the order of magnitude in regards to residential buildings in particular.) --Mr.98 (talk) 01:35, 30 October 2009 (UTC)
How accurate do you need? There are 300 million people in the USA, so I'd guess there are more than 5 million such buildings and less than 100 million. Dragons flight (talk) 02:00, 30 October 2009 (UTC)
To start, I tried Wolfram Search for "how many buildings in the US" and it didn't understand. (By comparison, it does answer how many people and how many dogs in the US, and includes a graph with the former question. The dogs get no graph.) Tempshill (talk) 03:15, 30 October 2009 (UTC)
If you did use the above link for new residential construction, you're still stuck with the problem of figuring out how many of those structures are still standing. Dismas|(talk) 04:57, 30 October 2009 (UTC)
Further, you have to define a building. Is a duplex a single building based on this question? There are apartments, condos, etc... We haven't touched on commercial and industrial buildings. Those can be complicated. Consider downtown Charleston, SC. Because of lack of space, it is common for the area (a wide alley) between two buildings on King Street to be filled in with another building. The result is a continuous wall along the street, but it is technically two buildings with a building crammed in between them. Is that three buildings or two buildings or one building? Further, I've been in a house there that is actually two houses that were close enough to be joined together. Is it one house now? -- kainaw 12:59, 30 October 2009 (UTC)
A house which is joined to another house is called semi-detached, they are very common in the UK. Lots of houses joined together side by side is a terraced house. Personally, I would define all those houses to be part of one building. I think that is how it is usually referred to in the UK. --Tango (talk) 00:46, 31 October 2009 (UTC)
The U.S. Census lists 78 million (+/- 0.5 million or so) houses with two or more stories compared to 41.5 million one-story ones.[22] But that doesn't include commercial buildings and if we had a figure for commercial buildings we would have to figure out how many were mixed use and counted twice. (also 78 million plus 41.5 million equals around 120 million - but the survey starts out telling us there are 128.2 million homes (+/- 48,000) so apparently 8 million houses have no floors?) The data is based on a survey of 53,000 addresses every other year. Rmhermen (talk) 13:35, 30 October 2009 (UTC)
I have seen this question on Ref Desk before, and the discussion there might be helpful, if a link to the appropriate archive could be found. Edison (talk) 15:38, 30 October 2009 (UTC)

Glycogen Storage in Human Muscle Cells[edit]

Hello. Where and how do the breakdown products of stored glycogen enter the cellular respiration pathway? Is glycogen stored in the vacuoles of the human muscle cells? Thanks in advance. --Mayfare (talk) 21:30, 29 October 2009 (UTC)

Its stored in bananas. 188.221.55.165 (talk) 22:10, 29 October 2009 (UTC)
Read the article on Glycogenesis. Glycogen is the form in which muscle cells can store glucose for immediate use. In the human brain it also play major role in cases where oxygen levels are very low and there is no other option for neurons to produce the energy they need to survive.--Gilisa (talk) 22:32, 29 October 2009 (UTC)
Perhaps glycogenolysis would be more apropos. DRosenbach (Talk | Contribs) 23:23, 29 October 2009 (UTC)
You are right. I didn't read his question correctly.--Gilisa (talk) 09:09, 30 October 2009 (UTC)

Does glycogenolysis occur in the vacuoles of human muscle cells? --Mayfare (talk) 01:57, 30 October 2009 (UTC)

Yes.--Gilisa (talk) 09:09, 30 October 2009 (UTC)

Ladybirds hibernating[edit]

Anyone know why ladybirds (or lady bugs if you are from the other place) hibernate in big groups rather than singly? It cannot reduce the risk of predators and I don't see warm as helpful but there are a few corners in our house which each year get several hundred bunched all winter all of which crawl off in the spring. Surely separate hibernation for a non colony insect makes more sense? Whats the advantage? --BozMo talk 22:25, 29 October 2009 (UTC)

For Coccinellidae being conspicuous is associated with reduced, rather than increased, predation; see Aposematism. --Dr Dima (talk) 23:12, 29 October 2009 (UTC)
Its because they secrete aggregation pheromones when they find a suitable overwintering site. Its not known how or why this happens, but it might be something to do with marking places that served them well the previous winter, on the basis that if the insects survived the winter there last year, they may do so again. (See also Insect winter ecology and Michael Majerus' Ladybird bible, ISBN 0-00-219935-1) Rockpocket 23:19, 29 October 2009 (UTC)
"Ladybugs practice communal hibernation by stacking one on top of one another on stumps and under rocks to share heat and buffer themselves against winter temperatures." Bus stop (talk) 23:29, 29 October 2009 (UTC)
Despite what our article states, its not quite as a simplistic as them aggregating together for warmth, since that could happen anywhere. Instead there appears to be a selection for special overwintering spots, suggesting there is location specific aggregation going on (see Pettersson et al, Eur. J. Entomol. 102: 365–370, 2005). Its been hypothesized that a non-volatile compound, 2-isopropyl-3-methoxypyrazine, is responsible (Abassi et al Cell. Mol. Life Sci. 54: 876–879), which additionally ensures that males and females are in the same locale when the breeding season comes around in Spring. Rockpocket 23:46, 29 October 2009 (UTC)


October 30[edit]

Collar-size of the woolly mammoth[edit]

Ok, I want to put a 16th century ruff about the neck of a woolly mammoth. How large would it have to be? Here's the dimensions from the WP article: they were not noticably taller than present-day Asian elephants, though they were heavier. Fully grown mammoth bulls reached heights between 2.8 m (9.2 ft) and 4.0 m (13 ft); the dwarf varieties reached between 1.8 m (5.9 ft) and 2.3 m (7.5 ft). They could weigh up to 8 tonnes.

Thanks Adambrowne666 (talk) 00:57, 30 October 2009 (UTC)

Is there some wooly mammoth neck diameter-mass-height ratio algorithm you know exists, but just don't know what it might be? Maybe figure one out based on the present day Asian elephant and fudge it -- ruff oglers will never know! DRosenbach (Talk | Contribs) 01:49, 30 October 2009 (UTC)
If you have access to a woolly mammoth for the dressing in a ruff, why can't you just use that access to also go in ahead of time to take a measurement? Dismas|(talk) 04:53, 30 October 2009 (UTC)
What? And spoil the surprise? Bielle (talk) 05:13, 30 October 2009 (UTC)
I was cheered up no end by thinking about woolly mammoths wearing ruffs! Readro (talk) 10:22, 30 October 2009 (UTC)
You will need Photoshop. Cuddlyable3 (talk) 17:37, 30 October 2009 (UTC)

The mammoth can be any size. The collar will have to be tailored to fit anyway.Cuddlyable3 (talk) 14:45, 31 October 2009 (UTC)

There are some museums which have life-size reconstructions of mammoths. Since there are real mammoths recovered in various states of decomposition from glaciers, these are likely to be fairly accurate, if recently constructed. I seem to recall a couple of such reconstructions outside the Cincinnati Museum of Natural History in early 1990, but the museum moved to new premises later that year, possibly discarding the mammoths, and these reconstructions are probably too old to be of use anyway.

Knee injury test[edit]

I was watching a documentary on football injuries. When testing for knee injuries, they did all the tests that I am used to seeing. Then, they did one that I don't understand. The person being tested took about one step forward with one foot. Keeping the back leg straight and bending the front leg, the person would apparently just shift his weight forward. Is this a standard test for a specific type of knee injury? If so, what is being tested? -- kainaw 03:19, 30 October 2009 (UTC)

Don't know specifically what the test is for, but that was a common exercise during physical therapy after my knee surgery. I know I did a lot of stuff related to the patella and making sure its accessory tendons and such were working to keep it properly centered, but I'm not sure if that specific exercise was related. — Lomn 12:29, 30 October 2009 (UTC)
It may very well be rehab and not a test. I don't know why I didn't think of that. Thanks. -- kainaw 12:41, 30 October 2009 (UTC)
I'm not aware of this as a specific test. However the manoeuvre causes tension on the quadriceps muscle and the quadriceps tendon (and to a lesser degree on the posterior cruciate ligament). It could be used as a test of quad muscle/tendon integrity. Axl ¤ [Talk] 19:47, 30 October 2009 (UTC)
This is ringing a very faint bell for me. Slap a "citation needed" tag on this, but ISTR that they're looking at whether the patella shifts (left-right) as the leg moves. I've been looking through this rather impressive list of knee tests, but don't see what you're describing or what I'm remembering. Could be a place to look through, though. Matt Deres (talk) 23:58, 31 October 2009 (UTC)

Jumping on concrete versus sand[edit]

When we jump on concrete we get more hurt then when we jump on sand!can we explain this by third law of motion? My sir told me 2answer according 2the seceond law of motion but i answered it according 2the third law saying that since the particles of sand r very fine compared to that of concrete they r not able 2offer enough resultant force as concrete does.....so we feel more hurt?

Is it correct? —Preceding unsigned comment added by 86.96.128.10 (talk) 10:06, 30 October 2009 (UTC)

Would it be the fineness of the sand particles, or the fact that they absorb more energy in the form of being displaced rather than posing a resilient barrier to the force and returning an equal an opposite force. It's like punching balloon (filled with air) or a punching bag. It's not that the air particles are smaller than whatever particle size composes the contents of a punching bag -- it's that the air moves away much more readily, allowing for greater dissipation of the imposed punching force. Unfortunately, though, I' well unaware of laws -- perhaps you can figure out to which law such a common sense approach would apply or be applied under. DRosenbach (Talk | Contribs) 12:21, 30 October 2009 (UTC)
You don't need any of Newton's laws of motion to explain this, although the second is the most applicable. The pain you feel is a result of the acceleration you're subject to when you land. Concrete doesn't give on impact, thus, your acceleration to a stop is very high (because the time for the change in velocity is very short). Per Newton's second law, that's a large force. However, if you jump onto loose sand (note the qualifier), it will shift around impact. That distributes your acceleration over a longer period of time, thus lowering its magnitude and the resultant force you feel. Note that jumping onto hard packed sand would be little different from hitting concrete. In any event, the difference is in how much whatever you impact moves. Newton's laws explain the peripheral factors (why time matters, why you hitting an object is like an object hitting you) but the core reason has nothing to do with them. — Lomn 12:24, 30 October 2009 (UTC)


(ec)I think that's going into way more detail than is justified. Here is a simple explanation:
  • -- Newton's second law says that the force you feel (which is what hurts!) is the mass times the acceleration. The mass is your body mass - but what is the acceleration?
  • -- Equations of motion says that the final velocity squared (which is zero because you end up stationary) equals initial velocity squared (the speed you hit the ground squared) plus twice the acceleration you're going to feel - times (s-si)...which is the distance over which you slowed down.
So, we can simplify that second equation to:
where d is the distance over which you slowed down to a stop.
In other words - the force you feel when you hit the ground is your body mass times the square of the speed you hit the ground at divided by twice the stopping distance...and that's the key here. We all know that it hurts more to hit the ground while wearing a heavy backpack (because your mass is larger) and it hurts a lot more the faster you are falling - but what about that stopping distance? A large stopping distance reduces your acceleration compared to a short one. Since landing on solid, immovable concrete forces your feet to slow down in just the distance that the soles of your shoes & feet can be compressed - you only get (let's say) 5 millimeters of stopping distance. But if you land in soft sand, you slow down as the sand gets pushed out of the way. If you leave a 10cm deep footprint - then your stopping distance was 100mm - plus the 5mm for the compression of the soles of your feet and your shoes. That's 21 times more stopping distance in the soft sand. And that means 21 times less force applied to your feet - and considerably less pain!
Of course in reality, it's not just your feet. Hopefully you didn't land with your legs straight and knees locked! Presuming your knees were bent a little then the upper part of your body can slow down over the distance that your knees bend on impact. That softens the blow to your vital organs and brain considerably more than the soft sand does.
This same principle explains why cars are designed with 'crumple zones'. A totally stiff, rigid car would stop in almost zero distance if you drove it into a brick wall. But a well-designed car is designed to crush and crumple selected bits of metal - which allows the car to slow down over a greater distance - hence less acceleration and less force on the poor passengers. The airbag fulfills a similar purpose in giving your head more distance to slow down over than if it hit the steering wheel.
You can explain this in other ways - such as how the energy of the impact is absorbed - but this explanation actually boils down to more or less the same thing - and it's much easier to understand than trying to predict the cohesion of sand grains in loose sand versus sand locked within a matrix of cement.
SteveBaker (talk) 12:25, 30 October 2009 (UTC)
See also Jerk (physics). While your force and acceleration are the same in the two systems, the Jerk is the relevent value that changes, and that changes how damaging the force is. --Jayron32 12:29, 30 October 2009 (UTC)
I beg to differ. The momentum lost is the same in both systems but the acc(dec)elerations are different. Cuddlyable3 (talk) 15:32, 30 October 2009 (UTC)
But it's not the loss of momentum that hurts! You lose the same amount of momentum falling into a big soft pile of feathers as you do smacking into a block of concrete - but the results are most certainly not equivalent. SteveBaker (talk) 16:20, 30 October 2009 (UTC)
Very true. That is why airbags are made for cars, and cute little mini airbags are made for mini cars. Cuddlyable3 (talk) 17:32, 30 October 2009 (UTC)
This question is of great interest to people designing things likes sports and dance floors and playgrounds and there are various standards e.g. EN 14904 for sports floors. Basically we have evolved to have the equivalent of shock absorbers like in a car in our feet and joints. Ours seem to be tuned to running on grass or sand and not to jumping on rocks like goats. This is bad for us as modern streets and buildings typically have concrete pavements and concrete floors with a sheet of vinyl on top. Doing anything except walking on these is dangerous and they can be lethal for the elderly if they fall over. Dmcq (talk) 14:09, 30 October 2009 (UTC)

I'm amazed at the different ways responders formulate a "simple" answer, but not that SteveBaker works a car into the answer. It would only be Wikisensation if he treated a mechanical question without a car. Below is my explanation.
When you jump on concrete it is like the problem of what happens when an irresistable force meets an immovable object? The answer is that at the moment of contact one of them has to give way. Either the concrete has to shatter or (more likely) the foot meeting the concrete has to decelerate to a stop very quickly. Newton's 2nd Law Force = mass x acceleration shows that when acceleration has a large value (deceleration is just a negative acceleration) and mass is the mass of your foot, Force will be large enough to hurt your foot. The rest of your body also decelerates but if you were sensible and kept your knees bent a little, its deceleration is less and continues after the feet have stopped.
When you jump on sand the difference is not just that the sand has small particles, it is also that there is space or water between the particles, allowing them relative movement. Your foot hits the uppermost grains of sand but they cannot resist with as much force as the concrete. The grains are pushed downwards, impacting and rubbing against the layer of grains below. The top layer of grains is now effectively part of your foot and the friction below them is a small Force that gives a small deceleration. So your foot plus sand layer continues downwards a little slower. That impacts the next sand layer layer and so on, and your foot comes to a full stop only after having pushed some distance into the sand. That is a big difference from the near-instantaneous deceleration on concrete. Although the momentum (mass x velocity) that you lose is the same in both falls, in the fall on sand the Force of deceleration has been spread in time so its maximum value is less hurtful. Jump down on to a mattress and the deceleration is even less and the experience not hurtful at all. Cuddlyable3 (talk) 15:28, 30 October 2009 (UTC)

I have always wondered, since the law of conservation of energy is true, once your foot hits the concrete, where does the energy form acceleration go?Accdude92 (talk to me!) (sign) 15:31, 30 October 2009 (UTC)
Heat, sound (which soon dissipates into more heat) and breaking stuff (pulling apart the bonds between the atoms of the material). SteveBaker (talk) 16:17, 30 October 2009 (UTC)
To the OP, your Sir was right. In fact all 3 of Newton's Laws can be seen at work during the jumps. Energy is conserved but there is good work-producing energy and lazy good-for-nothing workshy energy. Before you jump down on anything, consider that your potential energy is a Non-renewable resource whose expenditure causes by the Second law of thermodynamics an increase in Entropy thereby hastening the Heat death of the universe.Cuddlyable3 (talk) 17:20, 30 October 2009 (UTC)
Interestingly enough the 5mm that SteveBaker used as a minimum is actually about the very minimum deformation a floor should have to avoid the main danger of an old person breaking their hip if they fall over. It's quite small. a full centimeter is much better but I think it shows the way our ancestors evolved to just about be safe on the plains in Africa without spending too much resource on over protection. Dmcq (talk) 23:09, 30 October 2009 (UTC)
Yeah, the Savannah Hypothesis is pretty much disproved these days, as much as such a thing can be. It's tempting to make up just-so stories to explain observations about humans, but we should be cautious: what's the minimum deformation a floor should have to avoid the main danger of an old chimp breaking their hip if they fall over? What about a cat? What about a kangaroo? Is there any reason to believe these are significantly different? 86.139.237.128 (talk) 00:13, 31 October 2009 (UTC)
We have evolved to run and walk whatever about what hypothesis or collection of them that is under. If we needed to jump around rocks like goats we'd have evolved much better shock absorbers in our legs and joints. Chimps don't have as much problem as us on the ground because they aren't as tall and their bones are far stronger than needed just for support. Humans however are built more for lightness and endurance than strength. Dmcq (talk) 16:26, 31 October 2009 (UTC)

The Verity Incident[edit]

What was the Verity incident? Theallwordslinkedtalkman (talk) 16:27, 30 October 2009 (UTC)

Googling the exact phrase "Verity incident" only turns up 10 hits, and they aren't all for the same event. And verity can mean a lot of different things. So it'd be hard to answer this question without the context in which the phrase was used. Red Act (talk) 16:49, 30 October 2009 (UTC)
It was an incident where a fugitive was arrested at gunpoint at a school, reported here and here. Cuddlyable3 (talk) 17:28, 30 October 2009 (UTC)
Do you by chance mean the Vela Incident? Googlemeister (talk) 18:22, 30 October 2009 (UTC)

If I were small enough could I observe an atom?[edit]

If I were as small as an atom could I watch its operation? Presuming there is a reality down there, would the atom be observable by a hypothetical supertiny person? Such a person could obviously not exist so this is a thought experiment. What would we see given the extreme speeds of the particles, their being point-like and I believe the particle’s speeds and position are undetermined until measured. Thanks for any thoughts (I suspect the question has no answer!) - Adrian Pingstone (talk) 18:47, 30 October 2009 (UTC)

The biggest problem is that the wavelength of light is larger than the atom, so you couldn't see anything, including yourself, if you were that small. Otherwise, it would be possible to observe the movement of nuclei, and probably observe how the electron "clouds" behave in bonds, individual atoms, ions, etc. but due to the speed of the electrons' movements, you would be unable to observe individual electrons. The Seeker 4 Talk 19:13, 30 October 2009 (UTC)
Indeed, the problem is not that you are too big. The problem is that the resolution of light is inadequate. See "Microscopy", "Optical microscope" and "Electron microscope". The limit of resolution with light is about 200 nanometres. Axl ¤ [Talk] 20:00, 30 October 2009 (UTC)
These arguments are somewhat vague, as there exists near-field optics; also see the pages on near and far field, near-field scanning optical microscope and Near Field Communication (unfortunately, most of the articles are not very good, but you can check from journals that the idea works). The usual diffraction limits are calculated using the wave optics approximation to the Maxwell theory, an approximation that breaks up in the near-field case. Now on the original question: what should the hypothetical person consist of? Seeing means interaction between photons and the person's eyes, and the eyes' sensitivity depends on their composition.  Pt (T) 20:31, 30 October 2009 (UTC)
One thing you could do is feel individual atoms, using the principles used by the atomic force microscope and the scanning tunneling microscope. Note that individual atoms are visible in the images in those articles. Red Act (talk) 20:45, 30 October 2009 (UTC)
Not only that but if you were really that small, brownian motion would beat the heck out of you! You'd better hope it's really cold! SteveBaker (talk) 22:16, 30 October 2009 (UTC)
I think it's actually the Heisenberg uncertainty principle more than anything else that limits the ability to observe an atom on a fine scale -- it says that the more precisely you know the position of an object, the less precisely you can know its velocity. This basically means that regardless of your size you can't know the fine details of an atom's motion. Looie496 (talk) 23:03, 30 October 2009 (UTC)
You can see as precisely as you want, but you have to choose beforehand, what exactly you are going to look at. You can measure the position of a photon exactly, but then you cannot know the momentum (p) of the same photon and, as E==pc, you neither know its frequency (its colour). Thus you have to choose at least one of bad spatial resolution and colorblindness. The same applies for any other pair of observables corresponding to noncommuting operators in quantum mechanics. However, if you decided to measure only colours, it would definitely be an interesting picture. At another time you may as well watch the spatial dynamics of the photons reaching your eyes. Quantum mechanics does not make everything blurry, it just bites when you want to learn too much at a time!  Pt (T) 23:33, 30 October 2009 (UTC)
Actually, for such a small observer the whole concept of measurement changes as there the usual macroscopical decoherence does not happen anymore. The observer herself is a quantum object and we have no idea what a quantum consciousness in a notable superposition would sense. Note that the quantum mind is a different concept applied to try to explain the usual, macroscopical consciousness. It is all speculative indeed.  Pt (T) 00:46, 31 October 2009 (UTC)
You could not "see" things if you were a human as small as an atom, because you would be far smaller than a single rod or cone light receptor in the retina, and you would be far smaller than a single nerve cell in the human visual cortex. For the scheme to work, you would have to hypothesize that you and all your organs were made of atoms many orders of magnitude smaller than the atoms you were observing. The converse would be, "If an atom were as big as a bus, could I watch its operation?" In that case you might have to hypothesize the giant atom having physical constants such as Planck's constant and the strong and weak nuclear forces and electrical constants many orders of magnitude different than in our universe. Edison (talk) 01:16, 31 October 2009 (UTC)
You need little teeny eyes/for reading little teeny print/like you need little teeny license plates for bees. --Trovatore (talk) 01:18, 31 October 2009 (UTC)

As the question is phrased, you obviously meant for us to take it in a COMPLETELY non-literal fashion. Being so small, nothing in your body would work -- what would you consist of if you were small enough to see an atom? The lumen of your digestive system would be microscopic, as would the lumina of your ureters and your blood vessels. Even cells would be too large to traverse your circulatory system -- I mean, the ramifications are so many, it's somewhat ridiculous to even begin listing them here. I therefore take your question to mean, "Should an atom be able to be visualized in real time, would the observer be able to perceive the various motions ascribed to, for example, the orbiting motion of the electrons." For my purposes here, and hopefully for your purposes as well, just as I am ignoring all the infinite problems associated with the viewing organism being too small to possibly be able to exist, so too am I ignoring all integral problems related to light microscopy and the possibility of viewing items less than the diameter of a wavelength of light -- for certainly, you would not be interested in a similar response pertaining to adjunct obstacles in the vieweing of an atom, such as, "well, you wouldn't be able to view an atom because, if you were so small, you'd be in the circus and wouldn't have time to look at atoms. Thus said, perhaps the editors involved above can focus on such a question -- if you're still interested and my assumptions were correct. DRosenbach (Talk | Contribs) 17:27, 1 November 2009 (UTC)

This is the OP writing: DRosenbach is exactly correct, I was indeed imagining that I was actually down there at the atomic level with eyes and other senses that can still function. I would love to receive any more ideas - Adrian Pingstone (talk) 22:23, 1 November 2009 (UTC)

Small populations sizes and genetic mutations[edit]

When a tribe is discovered in the jungle or a new group of people are found on a remote island, I assume they have a greater amount of genetic drift which presumably results in more frequent mutations. Are there any recorded cases of a small population being discovered and all the individuals involved having some kind of beneficial mutation? I'm no scientist so please try and answer in a way I can understand.Popcorn II (talk) 19:33, 30 October 2009 (UTC)

There won't be more mutations; a mutation happens in an individual, so it doesn't matter what's happening in the rest of the population. Genetic drift will result in them having different allele proportions from the population they split off from. In a small population it, combined with the founder effect, will result in reduced genetic diversity. That means it is quite likely that a small isolated population will have every individual having a particular allele, and that allele could easily be a beneficial one. (Few alleles that aren't shared among all humans are beneficial to everyone, otherwise they would become shared among all individuals, but a certain allele may be beneficial in their environment - that is now evolution works.) --Tango (talk) 20:00, 30 October 2009 (UTC)
There are clear examples of adaptive characteristics found in isolated populations (though I'm not sure that any have been mapped to a single causative mutation). One potential example is the Moken, whose children have remarkable underwater vision. Its not known whether their superior underwater vision is a genetic or learned trait since one can learn to accommodate one's visual focus underwater, but rarely to the extent commonly seen in Moken children. But its certainly beneficial, given they spend much of their time diving for food. Rockpocket 20:24, 30 October 2009 (UTC)
The Moken's underwater visual abilities are probably learned [23]--Gilisa (talk) 22:54, 31 October 2009 (UTC)
Ah yes, it turns out their follow up study showed that other children can adapt just as well with the correct training (PMID 16806388). What a shame, when I first read their paper back in 2003, it hinted at a beautiful example of an adaptive genetic characteristic. Rockpocket 00:53, 1 November 2009 (UTC)

Flu vs. other diseases[edit]

How deadly is influenza compared to other infectious diseases? In particular, what other parasites cause similar numbers of deaths in, say, the U.S.? Thanks. 66.65.140.116 (talk) —Preceding undated comment added 20:05, 30 October 2009 (UTC).

According to the article Influenza vaccine, a report in 2008 cited that influenza accounted for about 41,000 deaths annually in the U.S. Worldwide figures can be found at Infectious disease#Mortality from infectious diseases, and influenza is counted there as part of a class called "lower respiratory infections", mixed in with things like pneumonia and stuff. So "flu-like" diseases are the largest cause of death from infectious agents worldwide, but I am not sure how this compares once you strip out the numbers for Influenza directly. Doing so would likely be impossible, since there are many non-influenza agents which cause nearly identical symptoms as influenza, and worldwide there is probably not the testing availible to seperate these. Diagnoses of "death from the flu" is probably made on a symptomatic basis, and as such, the best we can get on hard numbers would be "lower respiratory diseases". --Jayron32 20:18, 30 October 2009 (UTC)
If you're talking about mortality rate, flu is not terribly high on the list; it does get a high number of total deaths, though, in part because you can keep right on getting influenzas until one of them finally punches your ticket. Diseases with a very high rate of mortality include (in no particular order): HIV, Ebola, and untreated rabies (which, I think, was essentially at 100% until a small handful of people managed to pull through. Also, although their affects in humans may sometimes be quite similar, viruses and bacteria and generally not called parasites; that term is usually reserved for multicellular lifeforms. You may be interested in our article on List of causes of death by rate. Matt Deres (talk) 04:56, 1 November 2009 (UTC)

Tramadol expiry dates[edit]

Why do painkillers like tramadol carry an expiry date after which the instructions say you should not take them? Do they stop working? Or become dangerous? Or something else? Why? 86.166.155.90 (talk) 20:55, 30 October 2009 (UTC)

Generally the chemicals are not 100% stable and eventually break down, giving rise to byproducts which may or may not be harmful. Looie496 (talk) 21:04, 30 October 2009 (UTC)
Usually drugs just lose their effectiveness, but it is possible that the active ingredients, or one of the non-active ingredients that they use to make the bulk of the pill, will become potentially harmful. --Tango (talk) 09:32, 31 October 2009 (UTC)
About 2 yrs ago, I read an article on one of the common household NSAIDs and it mentioned that independent testing of the drug showed it was stable more than 2 years after the expiration date. The article suggested that expiration dates are as near as they are merely because the drug companies have only done testing for that long (e.g. 5 years). DRosenbach (Talk | Contribs) 00:02, 1 November 2009 (UTC)
A cynic might point out that keeping the period short sells more Advil. --Sean 13:41, 2 November 2009 (UTC)

October 31 onwards[edit]

Please see the current Science Reference Desk page